Ch 19 OB PrepU

¡Supera tus tareas y exámenes ahora con Quizwiz!

What would be the physiologic basis for a placenta previa?

low placental implantation

A woman with severe preeclampsia is receiving magnesium sulfate. The woman's serum magnesium level is 9.0 mEq/L. Which finding would the nurse most likely note?

diminished reflexes

A pregnant client with hyperemesis gravidarum needs advice on how to minimize nausea and vomiting. Which instruction should the nurse give this client?

Eat small, frequent meals throughout the day. The nurse should instruct the client with hyperemesis gravidarum to eat small, frequent meals throughout the day to minimize nausea and vomiting. The nurse should also instruct the client to avoid lying down or reclining for at least 2 hours after eating and to increase the intake of carbonated beverages. The nurse should instruct the client to try foods that settle the stomach such as dry crackers, toast, or soda.

A nurse is conducting a presentation for a group of pregnant women about conditions that can occur during pregnancy and that place the woman at high-risk. When discussing blood incompatibilities, which measure would the nurse explain as most effective in preventing isoimmunization during pregnancy?

Rho(D) immune globulin administration to Rh-negative women

A woman who is Rh negative asks the nurse how many children she will be able to have before Rh incompatibility causes them to die in utero. The nurse's best response would be that:

as long as she receives RhoGAM, there is no limit.

What would be the physiologic basis for a placenta previa?

low placental implantation The cause of placenta previa is usually unknown, but for some reason the placenta is implanted low instead of high on the uterus.

The nurse is developing a plan of care for a woman who is pregnant with twins. The nurse includes interventions focusing on which area because of the woman's increased risk?

preeclampsia

A pregnant woman has arrived to the office reporting vaginal bleeding. Which finding during the assessment would lead the nurse to suspect an inevitable abortion?

strong abdominal cramping

A woman in week 35 of her pregnancy with severe hydramnios is admitted to the hospital. The nurse recognizes that which concern is greatest regarding this client?

preterm rupture of membranes followed by preterm birth Even with precautions, in most instances of hydramnios, there will be preterm rupture of the membranes because of excessive pressure, followed by preterm birth. The other answers are not as big of concerns as preterm birth, in this situation.

A nurse is explaining to a group of nursing students that eclampsia or seizures in pregnant women are preceded by an acute increase in maternal blood pressure. Which of the following are features of an acute increase in blood pressure? Select all that apply. a) Auditory hallucinations b) Proteinuria c) Hypereflexia d) Blurring of vision e) Hyperglycemia

b) Proteinuria c) Hypereflexia d) Blurring of vision Rationale: Eclampsia is usually preceded by an acute increase in blood pressure as well as worsening signs of multi-organ system failure seen as increasing liver enzymes, proteinuria, and symptoms such as blurred vision and hyperreflexia. Hyperglycemia and auditory hallucinations are not seen with an acute increase in maternal BP.

A patient who is 16 weeks pregnant is passing pieces of body tissue along with blood clots and dark red blood from the vagina. What should the nurse direct the patient to do at this time?

Seek immediate medical attention and bring the expressed vaginal material.

Which of the following changes in B/P assessment findings during the second trimester indicate the highest risk for preeclampsia. a) Initial BP 120/80, current BP 130/88. b) Initial BP 110/60, current BP 112/86. c) Initial BP 100/70, current BP 140/90. d) Initial BP 140/85, current BP 130/80.

c) Initial BP 100/70, current BP 140/90. Rationale: A rise in blood pressure to above 140/90 is a concern the patient may be developing preeclampsia. The B/Ps noted in options A, C, and D are not indicative of developing preeclampsia, so these are incorrect responses.

A 32-year-old gravida 3 para 2 at 36 weeks' gestation comes to the obstetric department reporting abdominal pain. Her blood pressure is 164/90 mm/Hg, her pulse is 100 beats per minute, and her respirations are 24 per minute. She is restless and slightly diaphoretic with a small amount of dark red vaginal bleeding. What assessment should the nurse make next? a) Obtain a voided urine specimen and determine blood type. b) Measure fundal height. c) Check deep tendon reflexes. d) Palpate the fundus and check fetal heart rate.

d) Palpate the fundus and check fetal heart rate. Rationale: The classic signs of abruption placentea are pain, dark red vaginal bleeding, a rigid, board-like abdomen, hypertonic labor, and fetal distress.

The nurse is caring for a client with an ectopic pregnancy. Which symptom is a sign that the tube has ruptured?

Hypovolemic shock

A patient is admitted with a diagnosis of ectopic pregnancy. For what should the nurse anticipate preparing the patient?

Immediate surgery

Which compound would the nurse have readily available for a client who is receiving magnesium sulfate to treat severe preeclampsia?

calcium gluconate

A woman of 16 weeks' gestation telephones the nurse because she has passed some "berry-like" blood clots and now has continued dark brown vaginal bleeding. Which action would the nurse instruct the woman to do?

"Come to the health facility with any vaginal material passed."

A nurse in the maternity triage unit is caring for a client with a suspected ectopic pregnancy. Which nursing intervention should the nurse perform first?

Assess the client's vital signs.

A nurse in the maternity triage unit is caring for a client with a suspected ectopic pregnancy. Which nursing intervention should the nurse perform first?

Assess the client's vital signs. A suspected ectopic pregnancy can put the client at risk for hypovolemic shock. The assessment of vital signs should be performed first, followed by any procedures to maintain the ABCs. Providing emotional support would also occur, as would obtaining a surgical consent, if needed, but these are not first steps.

A pregnant client is being admitted for severe preeclampsia. When assigning room placement, which area would be most appropriate?

At the end of the hallway

The nurse is comforting and listening to a young couple who just suffered a miscarriage. When asked why this happened, which reason should the nurse share as a common cause?

Chromosomal abnormality

A client has been admitted to the hospital with a diagnosis of severe preeclampsia. Which nursing intervention is the priority?

Confine the client to bed rest in a darkened room.

A nurse is caring for a young woman who is in her 10th week of gestation. She comes into the clinic reporting vaginal bleeding. Which assessment finding best correlates with a diagnosis of hydatidiform mole?

Dark red, "clumpy" vaginal discharge

A woman at 37 weeks gestation presents to the labor and delivery area with symptoms of abruptio placentae. Which action should the nurse prioritize?

Ensure large bore IV access is obtained

The nurse is orientating in the Labor and Delivery unit and asks her preceptor how to differentiate a client with preeclampsia from one with eclampsia. Which symptoms would the preceptor describe to the new nurse as indicative of severe preeclampsia? Select all that apply.

Hyperactive deep tendon reflexes Nondependent edema Blood pressure above 160/110 mm Hg

The nurse is identifying nursing diagnoses for a patient with gestational hypertension. Which diagnosis would be the most appropriate for this patient?

Ineffective tissue perfusion related to vasoconstriction of blood vessels

The nurse is caring for a pregnant client with fallopian tube rupture. Which intervention is the priority for this client?

Monitor the client's vital signs and bleeding. A nurse should closely monitor the client's vital signs and bleeding (peritoneal or vaginal) to identify hypovolemic shock that may occur with tubal rupture. Beta-hCG level is monitored to diagnose an ectopic pregnancy or impending abortion. Monitoring the mass with transvaginal ultrasound and determining the size of the mass are done for diagnosing an ectopic pregnancy. Monitoring the FHR does not help to identify hypovolemic shock.

A client with a multiple gestation has come to a health care facility for a regular antenatal check-up. When educating the client on pregnancy, about which complication should the nurse inform the client?

Placental dysfunction

A woman in labor suddenly reports sharp fundal pain accompanied by slight dark red vaginal bleeding. The nurse should prepare to assist with which situation?

Premature separation of the placenta

A client is 20 weeks pregnant. At a prenatal visit, the nurse begins the prenatal assessment. Which finding would necessitate calling the primary care provider to assess the client?

The client has pink vaginal discharge and pelvic pressure.

A young mother gives birth to twin boys who shared the same placenta. What serious complication are they at risk for?

Twin-to-twin transfusion syndrome (TTTS)

The nurse is caring for a multigravid who experienced a placental abruption 4 hours ago. For which potential situation will the nurse prioritize assessment?

Uterine atony

The nurse is transcribing messages from the answering service. Which phone message should the nurse return first?

a 35-year-old, 21-week G3P2 client with blood pressure of 160/110 mm Hg, blurred vision, and whose last blood pressure was 143/99 mm Hg and urine dipstick showed a +2 proteinuria

The nurse is required to assess a pregnant client who is reporting vaginal bleeding. Which nursing action is the priority?

assessing the amount and color of the bleeding When the woman arrives and is admitted, assessing her vital signs, the amount and color of the bleeding, and current pain rating on a scale of 1 to 10 are the priorities. Assessing the signs of shock, monitoring uterine contractility, and determining the amount of funneling are not priority assessments when a pregnant woman complaining of vaginal bleeding is admitted to the hospital.

A pregnant client at 20 weeks' gestation arrives at the health care facility reporting excessive vaginal bleeding and no fetal movements. Which assessment finding would the nurse anticipate in this situation?

cervical incompetence

A woman develops HELLP syndrome. During labor, which of the following orders would you question? a) Assess her blood pressure every 15 minutes. b) Assess the urine output every hour. c) Urge her to lie on her left side during labor. d) Prepare her for epidural anesthesia.

d) Prepare her for epidural anesthesia. Rationale: A consequence of the HELLP syndrome is poor blood coagulation. Epidural anesthesia is not recommended when blood coagulation is in doubt.

A nurse is monitoring a client with PROM who is in labor and observes meconium in the amniotic fluid. What does the observation of meconium indicate?

fetal distress related to hypoxia

A client in her 20th week of gestation develops HELLP syndrome. What are features of HELLP syndrome? Select all that apply.

hemolysis elevated liver enzymes low platelet count

Which medication would the nurse prepare to administer if prescribed as treatment for an unruptured ectopic pregnancy?

methotrexate

A client for has an Rh-negative blood type. Following the birth of the client's infant, the nurse administers her Rho(D)immune globulin. The purpose of this is to:

prevent maternal D antibody formation. Because Rho(D) immune globulin contains passive antibodies, the solution will prevent the woman from forming long-lasting antibodies.

Which information on a client's health history would the nurse identify as contributing to the client's risk for an ectopic pregnancy?

recurrent pelvic infections

What is the drug methylergonovine maleate used for?

Uterine contraction

Upon entering the room of a client who has had a spontaneous abortion, the nurse observes the client crying. Which response by the nurse would be most appropriate?

"I'm sorry you lost your baby."

After teaching a woman who has had an evacuation for gestational trophoblastic disease (hydatidiform mole or molar pregnancy) about her condition, which statement indicates that the nurse's teaching was successful?

"I will be sure to avoid getting pregnant for at least 1 year."

The nurse understands the need to be aware of the potential of bleeding disorders in her pregnant clients. Which disorder should she be aware of that occurs in the second trimester?

Placenta previa

A 28-year-old client with a history of endometriosis presents to the emergency department with severe abdominal pain and nausea and vomiting. The client also reports her periods are irregular with the last one being 2 months ago. The nurse prepares to assess for which possible cause for this client's complaints?

Ectopic pregnancy

The nurse is caring for a pregnant client with fallopian tube rupture. Which intervention is the priority for this client?

Monitor the client's vital signs and bleeding.

A 24-year-old client is brought to the emergency department complaining of severe abodminal pain, vaginal bleeding, and fatigue. The nurse notes on assessment cool, clammy skin, confusion, and vital signs: HR 130, RR 28, and BP 98/60 mm Hg. Which action should the nurse prioritize?

Rule out shock.

Rhogam (Rh immune globulin) will be ordered for an RH - mother undergoing which of the following tests? a) Biophysical profile b) Contraction test c) Non-stress test d) Amniocentesis

d) Amniocentesis Rationale: Amniocentesis is a procedure requiring a needle to enter into the amniotic sac. There is a risk of mixing of the fetal and maternal blood which could result in blood incompatibility. A contraction test, a non-stress test, and biophysical profile are not invasive, so there would be no indication for Rhogam to be administered.

A pregnant client with severe preeclampsia has developed HELLP syndrome. In addition to the observations necessary for preeclampsia, what other nursing intervention is critical for this client?

observation for bleeding

The nurse is teaching a prenatal class on potential problems during pregnancy to a group of expectant parents. The risk factors for placental abruption are discussed. What comment validates accurate learning by the parents?

"Placental abruption is quite painful and I will need to let the doctor know if I begin to have abdominal pain."

The nurse is preparing the plan of care for a woman hospitalized for hyperemesis gravidarum. Which interventions would the nurse most likely include? Select all that apply.

1. maintaining NPO status for the first day or two 2. administering antiemetic agents 3. obtaining baseline blood electrolyte levels 4. monitoring intake and output

A pregnant client with multiple gestation arrives at the maternity clinic for a regular antenatal check up. The nurse would be aware that client is at risk for which perinatal complication?

congenital anomalies

A woman with an incomplete abortion is to receive misoprostol. The woman asks the nurse, "Why am I getting this drug?" The nurse responds to the client, integrating understanding that this drug achieves which effect?

ensures passage of all the products of conception Misoprostol is used to stimulate uterine contractions and evacuate the uterus after an abortion to ensure passage of all the products of conception. Rh (D) immunoglobulin is used to suppress the immune response and prevent isoimmunization.

A novice nurse asks to be assigned to the least complex antepartum client. Which condition would necessitate the least complex care requirements?

gestational hypertension

The nurse is preparing discharge instructions for a pregnant patient experiencing preterm rupture of membranes. What should the nurse include in this teaching? Select all that apply.

1. Avoid douching. 2. Measure oral temperature twice a day.

A client in her 20th week of gestation develops HELLP syndrome. What are features of HELLP syndrome? Select all that apply.

1. hemolysis 2. elevated liver enzymes 3. low platelet count

A primigravida 28-year-old client is noted to have Rh negative blood and her husband is noted to be Rh positive. The nurse should prepare to administer RhoGAM after which diagnostic procedure?

Amniocentesis

A 25-week-gestation client presents with a blood pressure of 152/99, pulse 78, no edema, and urine negative for protein. What would the nurse do next?

Notify the health care provider

The following hourly assessments are obtained by the nurse on a client with preeclampsia receiving magnesium sulfate: 97.3oF (36.2oC), HR 88, RR 1, BP 148/110 mm Hg. What other priority physical assessments by the nurse should be implemented to assess for potential toxicity?

Reflexes

The obstetric nurse is caring for a pregnant client who has been diagnosed with hydatidiform mole. What assessment should the nurse prioritize?

Vaginal bleeding

It is determined that a client's blood Rh is negative and her partner's is positive. To help prevent Rh isoimmunization, the nurse would expect to administer Rho(D) immune globulin at which time?

at 28 weeks' gestation and again within 72 hours after birth

A woman has been diagnosed as having pregnancy-induced hypertension. Which of the following is the most typical symptom of this? a) Increased perspiration b) Blood pressure elevation c) Weight loss d) Susceptibility to infection

b) Blood pressure elevation Rationale: The symptom of hypertension of pregnancy is blood pressure elevation (140/90 mm Hg).

A young mother delivers twin boys who shared the same placenta. What serious complication are they at risk for? a) TORCH syndrome b) ABO incompatability c) Twin-to-twin transfusion syndrome (TTTS) d) HELLP syndrome

c) Twin-to-twin transfusion syndrome (TTTS) Rationale: When twins share a placenta, a serious condition called twin-to-twin transfusion syndrome (TTTS) can occur.

After an examination, a client has been determined to have an unruptured ectopic pregnancy. Which medication would the nurse anticipate being prescribed?

methotrexate Methotrexate, a folic acid antagonist that inhibits cell division in the developing embryo, is most commonly used to treat ectopic pregnancy. Oxytocin is used to stimulate uterine contractions and would be inappropriate for use with an ectopic pregnancy. Promethazine and ondansetron are antiemetics that may be used to treat hyperemesis gravidarum.

A pregnant woman with preeclampsia is to receive magnesium sulfate IV. Which assessment should the nurse prioritize before administering a new dose?

patellar reflex

Which statement by a pregnant client indicates the need for more teaching about preeclampsia?

"If I have changes in my vision, I will lie down and rest." Changes in the visual field may indicate the client has moved from preeclampsia to severe preeclampisa and is at risk for developing a seizure due to changes in cerebral blood flow. The client would require immediate assessment and intervention. The other options would not indicate that more teaching about preeclampsia is indicated.

A nurse has been assigned to assess a pregnant client for abruptio placenta. For which classic manifestation of this condition should the nurse assess?

"knife-like" abdominal pain with vaginal bleeding The classic manifestations of abruption placenta are painful dark red vaginal bleeding, "knife-like" abdominal pain, uterine tenderness, contractions, and decreased fetal movement. Painless bright red vaginal bleeding is the clinical manifestation of placenta previa. Generalized vasospasm is the clinical manifestation of preeclampsia and not of abruptio placenta.

A student nurse asks the instructor what percentage of clinically recognized pregnancies end in miscarriages during the first trimester. Which response from the nurse is the most accurate?

15% to 20%

During pregnancy a woman's blood volume increases to accommodate the growing fetus to the point that vital signs may remain within normal range without showing signs of shock until the woman has lost what percentage of her blood volume?

40%

The nurse is assisting a client who has just undergone an amniocentesis. Blood results indicate the mother has type O blood and the fetus has type AB blood. The nurse should point out the mother and fetus are at an increased risk for which situation related to this procedure?

Baby developing postbirth jaundice

A pregnant patient is developing HELLP syndrome. During labor, which order should the nurse question?

Prepare for epidural anesthesia.

A nurse is preparing a nursing care plan for a client who is admitted at 22 weeks' gestation with advanced cervical dilatation to 5 cm, cervical insufficiency, and a visible amniotic sac at the cervical opening. Which primary goal should the nurse prioritize at this point?

Bed rest to maintain pregnancy as long as possible

A nurse is caring for a pregnant client with eclamptic seizure. Which is a characteristic of eclampsia?

Coma occurs after seizure. The nurse should know that coma usually follows an eclamptic seizure. Muscle rigidity occurs after facial twitching. Respirations do not become rapid during the seizure; they cease. Coma usually follows the seizure activity, with respiration resuming.

The nurse is caring for an Rh-negative nonimmunized client at 14 weeks' gestation. What information would the nurse provide to the client?

Obtain Rho(D) immune globulin at 28 weeks' gestation. The current recommendation is that every Rh-negative nonimmunized woman receives Rho(D) immune globulin at 28 weeks' gestation and again within 72 hours after giving birth. Consuming a well-balanced nutritional diet and avoiding sexual activity until after 28 weeks will not help to prevent complications of blood incompatibility. Transvaginal ultrasound helps to validate the position of the placenta and will not help to prevent complications of blood incompatibility.

A 32-year-old gravida 3 para 2 at 36 weeks' gestation comes to the obstetric department reporting abdominal pain. Her blood pressure is 164/90 mm/Hg, her pulse is 100 beats per minute, and her respirations are 24 per minute. She is restless and slightly diaphoretic with a small amount of dark red vaginal bleeding. What assessment should the nurse make next?

Palpate the fundus, and check fetal heart rate.

The nurse is caring for a client with preeclampsia and understands the need to auscultate this client's lung sounds every 2 hours. Why would the nurse do this?

Pulmonary edema

A client is 20 weeks pregnant. At a prenatal visit, the nurse begins the prenatal assessment. Which finding would necessitate calling the primary care provider to assess the client?

The client has pink vaginal discharge and pelvic pressure. Cervical dilatation usually occurs painlessly, and often the first symptom is pink vaginal discharge or increased pelvic pressure, which then is followed by rupture of membranes and discharge of the amniotic fluid. The other answers are nonthreatening signs and symptoms.

Vaginal bleeding during pregnancy is always a deviation from the normal.

True

A young woman presents at the emergency department with complaints of lower abdominal cramping and spotting at 12 weeks' gestation. The physician performs a pelvic examination and finds that the cervix is closed. What does the physician suspect is the cause of the cramps and spotting? a) Cervical insufficiency b) Threatened abortion c) Habitual abortion d) Ectopic pregnancy

b) Threatened abortion Rationale: Spontaneous abortion occurs along a continuum: threatened, inevitable, incomplete, complete, missed. The definition of each category is related to whether or not the uterus is emptied, or for how long the products of conception are retained.

A client in her 20th week of gestation develops HELLP syndrome. What are features of HELLP syndrome? Select all that apply.

hemolysis elevated liver enzymes low platelet count The HELLP syndrome is a syndrome involving hemolysis (microangiopathic hemolytic anemia), elevated liver enzymes, and a low platelet count. Hyperthermia and leukocytosis are not features of HELLP syndrome

During a routine prenatal visit, a client is found to have proteinuria and a blood pressure rise to 140/90 mm Hg. The nurse recognizes that the client has which condition?

mild preeclampsia

A client at 11 weeks' gestation experiences pregnancy loss. The client asks the nurse if the bleeding and cramping that occurred during the miscarriage were caused by working long hours in a stressful environment. What is the most appropriate response from the nurse?

"I can understand your need to find an answer to what caused this. Let's talk about this further."

A client at 11 weeks' gestation experiences pregnancy loss. The client asks the nurse if the bleeding and cramping that occurred during the miscarriage were caused by working long hours in a stressful environment. What is the most appropriate response from the nurse?

"I can understand your need to find an answer to what caused this. Let's talk about this further." Talking with the client may assist her to explore her feelings. She and her family may search for a cause for a spontaneous early bleeding so they can plan for future pregnancies. Even with modern technology and medical advances, however, a direct cause cannot usually be determined.

After a regular prenatal visit, a pregnant client asks the nurse to describe the differences between abruptio placenta and placenta previa. Which statement should the nurse include in the teaching?

"Placenta previa causes painless, bright red bleeding during pregnancy due to an abnormally implanted placenta that is too close to or covers the cervix; abruptio placenta is associated with dark red painful bleeding caused by premature separation of the placenta from the wall of the uterus before the end of labor." Placenta previa is a condition of pregnancy in which the placenta is implanted abnormally in the lower part of the uterus and is the most common cause of painless bright red bleeding in the third trimester. Abruptio placenta is the premature separation of a normally implanted placenta that pulls away from the wall of the uterus either during pregnancy or before the end of labor.

A 16-year-old client gave birth to a 12 weeks' gestation fetus last week. The client has come to the office for follow-up and while waiting in an examination room notices that on the schedule is written her name and "follow-up of spontaneous abortion." The client is upset about what is written on the schedule. How can the nurse best explain this terminology?

"Spontaneous abortion is a more specific term used to describe a spontaneous miscarriage, which is a loss of pregnancy before 20 weeks. This term does not imply that you did anything to affect the pregnancy." Abortion is a medical term for any interruption of a pregnancy before a fetus is viable, but it is better to speak of these early pregnancy losses as spontaneous abortions to avoid confusion with intentional terminations of pregnancies. The other responses are correct, but they do not provide the client with the most complete and reassuring answer.

A nurse is caring for a client with hyperemesis gravidarum. Which nursing action is the priority for this client?

Administer IV NS with vitamins and electrolytes. The first choice for fluid replacement is generally NS with vitamins and electrolytes added. If the client does not improve after several days of bed rest, "gut rest," IV fluids, and antiemetics, then total parenteral nutrition or percutaneous endoscopic gastrostomy tube feeding is instituted to prevent malnutrition.

A 17-year-old G2 P0010 patient develops hypertension at 32 weeks and is required to limit her intake of sodium and increase her protein intake. The patient is out of work and does not attend school. Her time is spent primarily at the local shopping mall with friends. The best dietary suggestion that the home visiting nurse can make to this patient is:

Bring water and a lunch from home rather than eating at the food court

A 32-year-old gravida 3 para 2 at 36 weeks' gestation comes to the obstetric department reporting abdominal pain. Her blood pressure is 164/90 mm/Hg, her pulse is 100 beats per minute, and her respirations are 24 per minute. She is restless and slightly diaphoretic with a small amount of dark red vaginal bleeding. What assessment should the nurse make next?

Palpate the fundus, and check fetal heart rate. The classic signs of abruptio placentae are pain, dark red vaginal bleeding, a rigid, board-like abdomen, hypertonic labor, and fetal distress.

A woman in labor is at risk for abruptio placentae. Which of the following assessments would most likely lead you to suspect that this has happened? a) Sharp fundal pain and discomfort between contractions. b) Painless vaginal bleeding and a fall in blood pressure. c) An increased blood pressure and oliguria. d) Pain in a lower quadrant and increased pulse rate.

a) Sharp fundal pain and discomfort between contractions. Rationale: An abruptio placentae refers to premature separation of the placenta from the uterus. As the placenta loosens, it causes sharp pain. Labor begins with a continuing nagging sensation. Painless vaginal bleeding and a fall in blood pressure are indicative of placenta previa. Pain in a lower quadrant and increased pulse rate are indicative of an ectopic pregnancy. Hypertension and oliguria are indicative of preeclampsia.

Vaginal bleeding during pregnancy is always a deviation from the normal. a) True b) False

a) True Rationale: Vaginal bleeding during pregnancy is always a deviation from the normal, is always potentially serious, may occur at any point during pregnancy, and is always frightening. It must always be carefully investigated because it can impair both the outcome of the pregnancy and the woman's life or health.

A woman who is Rh negative asks you how many children she will be able to have before Rh incompatibility causes them to die in utero. Your best response would be that a) she will have to ask her physician. b) no more than three children is recommended. c) only her next child will be affected. d) as long as she receives RhIG, there is no limit.

d) as long as she receives RhIG, there is no limit. Rationale: Because RhIG supplies passive antibodies, it prevents the woman from forming antibodies. Without antibodies that could affect the fetus, the woman could have as many children as she wants.

A woman is being closely monitored and treated for severe preeclampsia with magnesium sulfate. Which finding would alert the nurse to the development of magnesium toxicity in this client?

diminished reflexes

A nurse is reviewing the medical record of a pregnant client diagnosed with placenta previa. The physical exam reveals that the placenta is implanted near the internal os but does not reach it. The nurse interprets this as which type of placenta previa?

low-lying Placenta previa is generally classified according to the degree of coverage or proximity to the internal os, as follows: total placenta previa - the internal cervical os is completely covered by the placenta; partial placenta previa - the internal os is partially covered by the placenta; marginal placenta previa - the placenta is at the margin or edge of the internal os; low-lying placenta previa - the placenta is implanted in the lower uterine segment and is near the internal os but does not reach it.

During a routine prenatal visit, a client is found to have proteinuria and a blood pressure rise to 140/90 mm Hg. The nurse recognizes that the client has which condition?

mild preeclampsia A woman is said to have gestational hypertension when she develops an elevated blood pressure (140/90 mm Hg) but has no proteinuria or edema. If a seizure from gestational hypertension occurs, a woman has eclampsia, but any status above gestational hypertension and below a point of seizures is preeclampsia. A woman is said to be mildly preeclamptic when she has proteinuria and a blood pressure rise to 140/90 mm Hg, taken on two occasions at least 6 hours apart. A woman has passed from mild to severe preeclampsia when her blood pressure rises to 160 mm Hg systolic and 110 mm Hg diastolic or above on at least two occasions 6 hours apart at bed rest (the position in which blood pressure is lowest) or her diastolic pressure is 30 mm Hg above her prepregnancy level. Marked proteinuria, 3+ or 4+ on a random urine sample or more than 5 g in a 24-hour sample, and extensive edema are also present. A woman has passed into eclampsia when cerebral edema is so acute a grand-mal seizure (tonic-clonic) or coma has occurred.

A 44-year-old client has lost several pregnancies over the last 10 years. For the past 3 months, she has had fatigue, nausea, and vomiting. She visits the clinic and takes a pregnancy test; the results are positive. Physical examination confirms a uterus enlarged to 13 weeks' gestation; fetal heart tones are heard. Ultrasound reveals that the client is experiencing some bleeding. Considering the client's prenatal history and age, what does the nurse recognize as the greatest risk for the client at this time?

pregnancy loss

A 44-year-old client has lost several pregnancies over the last 10 years. For the past 3 months, she has had fatigue, nausea, and vomiting. She visits the clinic and takes a pregnancy test; the results are positive. Physical examination confirms a uterus enlarged to 13 weeks' gestation; fetal heart tones are heard. Ultrasound reveals that the client is experiencing some bleeding. Considering the client's prenatal history and age, what does the nurse recognize as the greatest risk for the client at this time?

pregnancy loss The client's advanced maternal age (pregnancy in a woman 35 years or older) increases her risk for pregnancy loss. Hypertension, preterm labor, and prematurity are risks as this pregnancy continues. Her greatest risk at 13 weeks' gestation is losing this pregnancy

A 24-year-old client presents in labor. The nurse notes there is an order to administer RhoGAM after the birth of her infant. When asked by the client the reason for this injection, which reason should the nurse point out?

prevent maternal D antibody formation.

The nurse is caring for a client with preeclampsia and understands the need to auscultate this client's lung sounds every two hours. Why would the nurse do this?

pulmonary edema In the hospital, monitor blood pressure at least every four hours for mild preeclampsia and more frequently for severe disease. In addition, it is important to auscultate the lungs every two hours. Adventitious sounds may indicate developing pulmonary edema.

When assessing a woman with an ectopic pregnancy, the nurse would suspect that the tube has ruptured based on which finding?

referred shoulder pain Referred pain to the shoulder area indicates bleeding into the abdomen caused by phrenic nerve irritation when a tubal pregnancy ruptures. Vaginal spotting, nausea, and breast tenderness are typical findings of early pregnancy and an unruptured ectopic pregnancy.

A pregnant woman has arrived to the office reporting vaginal bleeding. Which finding during the assessment would lead the nurse to suspect an inevitable abortion?

strong abdominal cramping Strong abdominal cramping is associated with an inevitable spontaneous abortion. Slight vaginal bleeding early in pregnancy and a closed cervical os are associated with a threatened abortion. With an inevitable abortion, passage of the products of conception may occur. No fetal tissue is passed with a threatened abortion.

A woman in week 16 of her pregnancy calls her primary care provider's office to report that she has experienced abdominal cramping, cervical dilation, vaginal spotting, and the passing of tissue. The nurse instructs the client to bring the passed tissue to the hospital with her. What is the correct rationale for this instruction?

to determine whether gestational trophoblastic disease is present Gestational trophoblastic disease is abnormal proliferation and then degeneration of the trophoblastic villi. The embryo fails to develop beyond a primitive start. Abnormal trophoblast cells must be identified because they are associated with choriocarcinoma, a rapidly metastasizing malignancy. This is why it is important for any woman who begins to miscarry at home to bring any clots or tissue passed to the hospital with her. The presence of clear fluid-filled cysts changes the diagnosis from a simple miscarriage to gestational trophoblastic disease. The client is not instructed to bring in passed tissue to determine whether infection is present or the fetus is viable or to determine the stage of development of the fetus.

A nurse is conducting a presentation for a group of pregnant women about conditions that can occur during pregnancy and that place the woman at high-risk. When discussing blood incompatibilities, which measure would the nurse explain as most effective in preventing isoimmunization during pregnancy?

Rho(D) immune globulin administration to Rh-negative women Rh incompatibility can be prevented with the use of Rho(D) immune globulin. Hemolysis associated with ABO incompatibility is limited to mothers with type O blood and their fetuses with type A or B blood. Amniocentesis would be appropriate for treatment of polyhydramnios, not isoimmunization. Cerclage is a treatment for cervical insufficiency.

A nurse is describing the use of Rho(D) immune globulin as the therapy of choice for isoimmunization in Rh-negative women and for other conditions to a group of nurses working at the women's health clinic. The nurse determines that additional teaching is needed when the group identifies which situation as an indication for Rho(D) immune globulin?

STIs Indications for Rho(D) immune globulin include isoimmunization, ectopic pregnancy, chorionic villus sampling, amniocentesis, prenatal hemorrhage, molar pregnancy, maternal trauma, percutaneous umbilical sampling, therapeutic or spontaneous abortion, fetal death, or fetal surgery.

A female client presents to the emergency department reporting she recently had a positive home pregnancy test but is now suspected of having an ectopic pregnancy. Which assessment should the nures prioritize?

Shoulder pain

The nurse is caring for a client who has a multifetal pregnancy. What topic should the nurse prioritize during health education?

Signs of preterm labor

A 35-year-old client is seen for her 2-week postoperative appointment after a suction curettage was performed to evacuate a hydatidiform mole. The nurse explains that the human chorionic gonadotropin (hCG) levels will be reviewed every 2 weeks and teaches about the need for reliable contraception for the next 6 months to a year. The client states, "I'm 35 already. Why do I have to wait that long to get pregnant again?" What is the nurse's best response?

"A contraceptive is used so that a positive pregnancy test resulting from a new pregnancy will not be confused with the increased level of hCG that occurs with a developing malignancy." Because of the risk of choriocarcinoma, the woman receives extensive treatment. Therapy includes baseline chest X-ray to detect lung metastasis physical exam including pelvic exam. Serum B-hCG levels weekly until negative results are obtained three consecutive times, then monthly for 6 to 12 months. The woman is cautioned to avoid pregnancy during this time because the increasing B-hCG levels associated with pregnancy would cause confusion as to whether cancer had developed. If after a year B-hCG seruim titers are within normal levels, a normal pregnancy can be achieved.

A woman of 16 weeks' gestation telephones the nurse because she has passed some "berry-like" blood clots and now has continued dark brown vaginal bleeding. Which action would the nurse instruct the woman to do?

"Come to the health facility with any vaginal material passed." This is a typical time in pregnancy for gestational trophoblastic disease to present. Asking the woman to bring any material passed vaginally would be important so it can be assessed for this.

A pregnant client at 32 weeks' gestation calls the clinic and informs the nurse that she thinks her membranes are leaking. She states that some clear fluid has run down her leg. What is the best response by the nurse?

"It is best for you to visit a hospital immediately. They can use a nitrazine strip to determine if it is amniotic fluid." The practitioner will perform a speculum examination, looking for pooling of amniotic fluid, and then test the fluid with nitrazine paper, which turns blue in the presence of amniotic fluid. Preterm premature rupture of membranes occurs when the rupture of the amniotic sac before the onset of labor happens in a woman who is less than 37 weeks' gestation.

A client with a history of cervical insufficiency is seen for reports of pink-tinged discharge and pelvic pressure. The primary care provider decides to perform a cervical cerclage. The nurse teaches the client about the procedure. Which client response indicates that the teaching has been effective?

"Purse-string sutures are placed in the cervix to prevent it from dilating."

The client arrives in the office and reports that her feet and legs swelling. During a client evaluation, the nurse notes that she can elicit a 4-mm skin depression that disappears in 10 to 15 seconds. The nurse correctly documents this finding as:

2+ pitting edema. Pitting edema is recorded using the following relative scale: 1+ is a 2-mm depression that disappears rapidly; 2+ is a 4-mm depression that disappears in 10 to 15 seconds; 3+ is a 6-mm depression that lasts more than one minute; and 4+ is an 8-mm depression that lasts 2 to 3 minutes.

A pregnant client is admitted to a health care unit with disseminated intravascular coagulation (DIC). Which prescription is the nurse most likely to receive regarding the therapy for such a client?

Administer cryoprecipitate and platelets.

During pregnancy a woman's blood volume increases to accommodate the growing fetus to the point that vital signs may remain within normal range without showing signs of shock until the woman has lost what percentage of her blood volume?

40% Vital signs can be within normal range, even with significant blood loss, because a pregnant woman can lose up to 40% of her total blood volume without showing signs of shock.

A pregnant patient is diagnosed with placenta previa. Which action should the nurse implement immediately for this patient?

Assess fetal heart sounds with an external monitor.

A woman in her 20s has experienced a miscarriage at 10 weeks' gestation and asks the nurse at the hospital what went wrong. She is concerned that she did something that caused her to lose her baby. The nurse can reassure the woman by explaining that the most common cause of spontaneous miscarriage in the first trimester is related to which factor?

Chromosomal defects in the fetus

Current research indicates that supplementation with what before pregnancy may reduce the risk of placental abruption?

Folic acid

A 25-year-old client at 22 weeks' gestation is noted to have proteinuria and dependent edema on her routine prenatal visit. Which additional assessment should the nurse prioritize and alert the RN or health care provider?

Initial BP 100/70 mm Hg; current BP 140/90 mm Hg

A pregnant woman has been admitted to the hospital due to severe preeclampsia. Which measure will be important for the nurse to include in the care plan?

Institute and maintain seizure precautions.

A pregnant woman has been admitted to the hospital due to severe preeclampsia. Which measure will be important for the nurse to include in the care plan?

Institute and maintain seizure precautions. The woman with severe preeclampsia should be maintained on complete bed rest in a dark and quiet room to avoid stimulation. The client is at risk for seizures; therefore, institution and maintenance of seizure precautions should be in place.

A patient recovering from an uneventful vaginal delivery is prescribed Rho(D) immune globulin (RhIG). What should the nurse explain to the patient regarding the purpose of this medication?

It prevents maternal D antibody formation.

The nurse is caring for a pregnant client with severe preeclampsia. Which nursing intervention should a nurse perform to institute and maintain seizure precautions in this client?

Keep the suction equipment readily available. The nurse should institute and maintain seizure precautions such as padding the side rails and having oxygen, suction equipment, and call light readily available to protect the client from injury. The nurse should provide a quiet, darkened room to stabilize the client. The nurse should maintain the client on complete bed rest in the left lateral lying position and not in a supine position. Keeping the head of the bed slightly elevated will not help maintain seizure precautions.

Spontaneous miscarriage occurs in 5% to 10% of all pregnancies. a) False b) True

a) False Rationale: Spontaneous miscarriage occurs in 15% to 30% of all pregnancies and arises from natural causes.

The nurse is required to assess a pregnant client who is reporting vaginal bleeding. Which nursing action is the priority?

assessing the amount and color of the bleeding

A client at 37 weeks' gestation presents to the emergency department with a BP 150/108 mm Hg, 1+ pedal edema, 1+ proteinuria, and normal deep tendon reflexes. Which assessment should the nurse prioritize as the client is administered magnesium sulfate IV?

Respiratory rate

A nurse is working as part of a community group focusing efforts on preventing isoimmunization during pregnancy at the local women's health clinic. Which measure would the group encourage?

Rho(D) immune globulin administration to Rh-negative women Rh incompatibility can be prevented with the use of Rho(D) immune globulin. Hemolysis associated with ABO incompatibility is limited to mothers with type O blood and their fetuses with type A or B blood. Amniocentesis would be appropriate for treatment of polyhydramnios not isoimmunization. Cerclage is a treatment for cervical insufficiency.

The nurse is concerned that a pregnant patient is experiencing abruptio placentae. What did the nurse assess in this patient?

Sharp fundal pain and discomfort between contractions

A pregnant patient with a history of premature cervical dilatation undergoes cervical cerclage. Which outcome indicates that this procedure has been successful?

The client delivers a full-term fetus at 39 weeks' gestation.

A woman and her partner present at her first antenatal obstetrics appointment. She is 6 weeks pregnant. Her blood tests show that she is Rh negative, although she has not previously known this. She has no other children but suspects that she miscarried early in a pregnancy 2 years before. She felt fine afterward, so she never received medical attention for that suspected episode. Her partner reports that he is also Rh negative. What action will the nurse be likely to take?

The nurse will perform all normal procedures and follow-up tasks because the Rh status of the client and the child are not a concern at this time. If the woman is Rh negative and her partner is Rh negative, the fetus will also be Rh negative, and the woman will not require treatment with Rho(D) immune globulin. Therefore, the remaining options are incorrect.

A woman in labor is at risk for abruptio placentae. Which assessment would most likely lead the nurse to suspect that this has happened?

sharp fundal pain and discomfort between contractions

A pregnant woman is being evaluated for HELLP. The nurse reviews the client's diagnostic test results. Which result would the nurse interpret as helping to confirm this diagnosis?

elevated LDH HELLP (hemolysis, elevated liver enzymes, low platelet count) syndrome is a variant of the preeclampsia/eclampsia syndrome. The diagnosis is based on laboratory test results, including: low hematocrit, elevated LDH, elevated AST, elevated ALT, elevated BUN, elevated bilirubin level, elevated uric acid and creatinine levels, and low platelet count. White blood cell counts are not used to evaluate for HELLP.

The nurse is assessing a client at 12 weeks' gestation at a routine prenatal visit who reports something doesn't feel right. Which assessment findings should the nurse prioritize?

gestational hypertension, hyperemesis gravidarum, absence of FHR

A nurse is providing care to a client who has been diagnosed with a common benign form of gestational trophoblastic disease. The nurse identifies this as:

hydatidiform mole.

A woman in labor has sharp fundal pain accompanied by slight vaginal bleeding. What would be the most likely cause of these symptoms?

premature separation of the placenta

A woman at 8 weeks' gestation is admitted for ectopic pregnancy. She is asking why this has occurred. The nurse knows that which factor is a known risk factor for ectopic pregnancy?

use of IUD for contraception

A client suffering a miscarriage at 12 weeks' gestations is very upset that the health care provider has ordered a D&C. How should the nurse respond after the client states she didn't have a D&C the time she lost a previous baby at 5 weeks' gestation?

"This procedure is needed to adequately remove all the fetal tissue."

Which nursing diagnosis would be most appropriate for a woman diagnosed with gestational hypertension?

Deficient fluid volume related to vasospasm of arteries Gestational hypertension is caused by vascular spasm. This leads to increased blood pressure and edema. Extensive edema leads to a deficiency of fluid volume.

A client is diagnosed with gestational hypertension and is receiving magnesium sulfate. Which finding would the nurse interpret as indicating a therapeutic level of medication?

deep tendons reflexes 2+

A nurse is conducting a refresher program for a group of perinatal nurses. Part of the program involves a discussion of HELLP. The nurse determines that the group needs additional teaching when they identify which aspect as a part of HELLP?

elevated lipoproteins The acronym HELLP represents hemolysis, elevated liver enzymes, and low platelets. This syndrome is a variant of preeclampsia/eclampsia syndrome that occurs in 10% to 20% of clients whose diseases are labeled as severe.

A woman is admitted with a diagnosis of ectopic pregnancy. For which procedure should the nurse prepare?

immediate surgery Ectopic pregnancy means an embryo has implanted outside the uterus, usually in the fallopian tube. Surgery is usually necessary to remove the growing structure before the tube ruptures or to repair the tube if rupture has occurred already.

A pregnant client has an Rh-negative blood type. Following the birth of the client's infant, the nurse administers her Rho(D) immune globulin. The purpose of this is to:

prevent maternal D antibody formation.

A woman at 8 weeks' gestation is admitted for ectopic pregnancy. She is asking why this has occurred. The nurse knows that which factor is a known risk factor for ectopic pregnancy?

use of IUD for contraception Use of an IUD with progesterone has a known increased risk for development of ectopic pregnancies. The nurse needs to complete a full history of the client to determine if she had any other risk factors for an ectopic pregnancy. Adhesions, scarring, and narrowing of the tubal lumen may block the zygote's progress to the uterus. Any condition or surgical procedure that can injure a fallopian tube increases the risk. Examples include salpingitis, infection of the fallopian tube, endometriosis, history of prior ectopic pregnancy, any type of tubal surgery, congenital malformation of the tube, and multiple elective abortions. Conditions that inhibit peristalsis of the tube can result in tubal pregnancy. Hormonal factors may play a role because tubal pregnancy occurs more frequently in women who take fertility drugs or who use progesterone intrauterine contraceptive devices (IUDs). A high number of pregnancies, multiple gestation pregnancy, and the use of oral contraceptives are not known risk factors for ectopic pregnancy.

Which is the best question the nurse can ask a woman who is leaving the hospital after experiencing a complete spontaneous abortion?

"Do you have someone to talk to, or may I give you the names and numbers for some possible grief counselors?" When a woman has a spontaneous abortion one important consideration is the emotional needs of the woman once she is home. She may not want to talk about the loss for a period of time, but the nurse needs to determine her support system for the future. Asking the woman if she is "going to try again" is an inappropriate question for the nurse to ask and diminishes the experience of having a spontaneous abortion. Giving the woman statistical information on spontaneous abortions is not appropriate when this client needs support and caring concern. Offering to give the client resources to aid in smoking cessation is not addressed in the scenario, so this is an inappropriate response.

After a regular prenatal visit, a pregnant client asks the nurse to describe the differences between abruptio placentae and placenta previa. Which statement should the nurse include in the teaching?

"Placenta previa causes painless, bright red bleeding during pregnancy due to an abnormally implanted placenta that is too close to or covers the cervix; abruptio placentae is associated with dark red painful bleeding caused by premature separation of the placenta from the wall of the uterus before the end of labor."

A client with a history of cervical insufficiency is seen for reports of pink-tinged discharge and pelvic pressure. The primary care provider decides to perform a cervical cerclage. The nurse teaches the client about the procedure. Which client response indicates that the teaching has been effective?

"Purse-string sutures are placed in the cervix to prevent it from dilating." The cerclage, or purse string suture is inserted into the cervix to prevent preterm cervical dilatation and pregnancy loss. Staples, glue, or a cervical cap will not prevent the cervix from dilating.

A client in her first trimester arrives at the emergency room with reports of severe cramping and vaginal spotting. On examination, the health care provider informs her that no fetal heart sounds are evident and orders a dilatation and curettage. The client looks frightened and confused and states that she does not believe in abortion. Which statement by the nurse is best?

"Unfortunately, the pregnancy is already lost. The procedure is to clear the uterus to prevent further complications."

A client is 11 weeks pregnant after many years trying to conceive. After arriving home from a normal prenatal visit, she experiences mild cramping and has a gush of bright red vaginal bleeding. She calls the nurse and reports having soaked a pad with fresh blood in fewer than 30 minutes. The uterine cramping is worsening. What is the most appropriate response from the nurse?

"You need to seek immediate attention from the primary care provider."

A nurse has been assigned to assess a pregnant client for abruptio placenta. For which classic manifestation of this condition should the nurse assess?

"knife-like" abdominal pain with vaginal bleeding

The nurse is monitoring a pregnant patient who is receiving intravenous magnesium sulfate for eclampsia. During the last assessment, the nurse was unable to elicit a patellar reflex. What should the nurse do?

Stop the current infusion.

The nurse is caring for a woman at 32 weeks gestation with severe preeclampsia. Which assessment finding should the nurse prioritize after the administration of hydralazine to this client?

Tachycardia

A 25-year-old pregnant client has just been diagnosed with hyperemesis gravidarum. Which instruction should the nurse prioritize during a teaching session?

Take your anti-nausea medicine around the clock.

The nurse is transcribing messages from the answering service. Which phone message should the nurse return first?

a 35-year-old, 21-week G3P2 client with blood pressure of 160/110 mm Hg, blurred vision, and whose last blood pressure was 143/99 mm Hg and urine dipstick showed a +2 proteinuria The nurse should call the at-risk 35-year-old client first. She is 21 weeks and has symptoms (blurred vision) of preeclampsia. She also had an increase of protein in her urine (2+) and a 15% increase in her BP. The nurse will need more information to determine if the 38-week client may be in the early stages of labor, and if the 31-week client with flank pain has a kidney infection. The client with malaise and rhinitis will need to talk to the nurse last to find out what over-the-counter medication she is able to take.

A primipara at 36 weeks' gestation is being monitored in the prenatal clinic for risk of preeclampsia. Which sign or symptom is the priority concern for the nurse?

a dipstick value of 2+ for protein The increasing amount of protein in the urine is a concern the preeclampsia may be progressing to severe preeclampsia. The woman needs further assessment by the health care provider. Dependent edema may be seen in the majority of pregnant women and is not an indicator of progression from preeclampsia to eclampsia. Weight gain is no longer considered an indicator for the progression of preeclampsia. A systolic blood pressure increase is not the highest priority concern for the nurse, since there is no indication what the baseline blood pressure was

A client at 11 weeks' gestation experiences pregnancy loss. The client asks the nurse if the bleeding and cramping that occurred during the miscarriage were caused by working long hours in a stressful environment. What is the most appropriate response from the nurse? a) "I can understand your need to find an answer to what caused this. Let's talk about this further." b) "It is hard to know why a woman bleeds during early pregnancy." c) "Your spontaneous bleeding is not work-related." d) "Something was wrong with the fetus."

a) "I can understand your need to find an answer to what caused this. Let's talk about this further." Rationale: Talking with the client may assist her to explore her feelings. She and her family may search for a cause for a spontaneous early bleeding so they can plan for future pregnancies. Even with modern technology and medical advances, however, a direct cause cannot usually be determined.

A client experiences a threatened abortion. She is concerned about losing the pregnancy and asks what activity level she should maintain. What is the most appropriate response from the nurse? a) "Restrict your physical activity to moderate bedrest." b) "There is no research evidence that I can recommend to you." c) "Carry on with the activity you engaged in before this happened." d) "Strict bedrest is necessary so as not to jeopardize this pregnancy."

a) "Restrict your physical activity to moderate bedrest." Rationale: With a threatened abortion, moderate bedrest and supportive care are recommended. Regular physical activity may increase the chances of miscarriage. Strict bedrest is not necessary. Activity restrictions are part of standard medical management.

A student nurse asks the instructor what percentage of clinically recognized pregnancies end in miscarriages during the first trimester. The most accurate response from the registered nurse is which of the following? a) 11% to 20% b) 21% to 30% c) 5% to 10% d) 31% to 40%

a) 11% to 20% Rationale: During the first trimester, 10% to 20% of all clinically recognized pregnancies end in miscarriage.

A primipara at 36 weeks gestation is being monitored in the prenatal clinic for risk of preeclampsia. Which of the following signs or symptoms is the priority concern for the nurse? a) A dipstick value of 2+ for protein. b) Weight gain of 1.2 lb during the past 1 week. c) A systolic blood pressure increase of 10 mm hg. d) Pedal edema.

a) A dipstick value of 2+ for protein. Rationale: The increasing amount of protein in the urine is a concern the preeclampsia may be progressing to severe preeclampsia. The woman needs further assessment by the health care provider. Dependent edema may be seen in the majority of pregnant women and is not an indicator of progression from preeclampsia to eclampsia. Weight gain is no longer considered an indicator for the progression of preeclampsia. A systolic blood pressure increase is not the highest priority concern for the nurse, since we have no idea what the baseline blood pressure was.

A pregnant client is admitted to a health care unit with disseminated intravascular coagulation (DIC). Which of the following orders is the nurse most likely to receive regarding the therapy for such a client? a) Administer cryoprecipitate and platelets b) Give each unit of blood to raise the hematocrit by 3 g/dL c) Administer a ratio of 1 unit of blood to 4 units of frozen plasma d) Aim at keeping the client's hematocrit above 20%

a) Administer cryoprecipitate and platelets. Rationale: In a pregnant client with DIC, the nurse may be ordered to administer cryoprecipitate and platelets. Whole blood does not contain clotting factors. Therefore a ratio of 4 units of blood to 1 unit of fresh frozen plasma, and not 1 unit of blood to 4 units of frozen plasma, should be considered. The nurse should aim at maintaining the client's hematocrit above 30% and not just 20%. The nurse should expect one unit of blood to increase the hematocrit by 1.5 g/dL, and not 3g/dL.

A patient is admitted at 22 weeks gestation with advanced cervical dilatation to 5 centimeters, cervical insufficiency, and a visible amniotic sac at the cervical opening. What is the primary goal for this patient at this point? a) Bed rest to maintain pregnancy as long as possible b) Deliver vaginally c) Notification of social support for loss of pregnancy d) Education on causes of cervical insufficiency for the future

a) Bed rest to maintain pregnancy as long as possible Rationale: At 22 weeks gestation, the fetus is not viable. The woman would be placed on bed rest, total, with every attempt made to halt any further progression of dilatation as long as possible. You would not want to deliver this fetus vaginally at this stage of gestation. It is not your responsibility to notify the patient's social support of a possible loss of the pregnancy. It is not appropriate at this time to educate the mother on causes of cervical insufficiency for future pregnancies.

You are caring for a young woman who is in her 10th week of gestation. She comes into the clinic complaining of vaginal bleeding. Which assessment finding best correlates with a diagnosis of hydatidiform mole? a) Dark red, "clumpy" vaginal discharge b) Painful uterine contractions and nausea c) Bright red painless vaginal bleeding d) Brisk deep tendon reflexes and shoulder pain

a) Dark red, "clumpy" vaginal discharge Rationale: If a complete molar pregnancy continues into the second trimester undetected, other signs and symptoms appear. The woman often presents with complaints of dark to bright red vaginal bleeding and pelvic pain. Infrequently, she will report passage of grapelike vesicles.

A client reports to her obstetrician complaining of a significant amount of bright red, painless vaginal bleeding. A sonogram reveals that her placenta has implanted low in the uterus and is partially covering the cervical os. Which of the following immediate care measures should the nurse initiate? (Select all that apply.) a) Determine from the client the time the bleeding began and about how much blood has been lost b) Attach external monitoring equipment to record fetal heart sounds c) Continue to assess blood pressure every 5 to 15 minutes d) Perform a pelvic examination e) Place the woman on bed rest in a side-lying position f) Obtain baseline vital signs

a) Determine from the client the time the bleeding began and about how much blood has been lost b) Attach external monitoring equipment to record fetal heart sounds c) Continue to assess blood pressure every 5 to 15 minutes e) Place the woman on bed rest in a side-lying position f) Obtain baseline vital signs Rationale: With the exception of performing a pelvic examination, all of the answers are appropriate immediate care measures for the client with placenta previa. Never attempt a pelvic or rectal examination with painless bleeding late in pregnancy because any agitation of the cervix when there is a placenta previa might tear the placenta further and initiate massive hemorrhage, possibly fatal to both mother and child.

A client in her 20th week of gestation develops HELLP syndrome. Which of the following should the nurse consider as features of HELLP syndrome? Select all that apply. a) Elevated liver enzymes b) Leukocytosis c) Low platelet count d) Hemolysis e) Hyperthermia

a) Elevated liver enzymes c) Low platelet count d) Hemolysis Rationale: The HELLP syndrome is a syndrome involving hemolysis (microangiopathic hemolytic anemia), elevated liver enzymes, and a low platelet count. Hyperthermia and leukocytosis are not features of HELLP syndrome.

A woman with an incomplete abortion is to receive misoprostol. The nurse understands that the rationale for administering this drug is to: a) Ensure passage of all the products of conception b) Suppress the immune response to prevent isoimmunization c) Halt the progression of the abortion d) Alleviate strong uterine cramping

a) Ensure passage of all the products of conception Rationale: Misoprostol is used to stimulate uterine contractions and evacuate the uterus after an abortion to ensure passage of all the products of conception. Rh (D) immunoglobulin is used to suppress the immune response and prevent isoimmunization.

Which of the following would the nurse most likely include when planning the care for a woman requiring hospitalization for hyperemesis gravidarum? Select all that apply. a) Maintaining NPO status for the first day or two b) Administering antiemetic agents c) Preparing the woman for insertion of a feeding tube d) Obtaining baseline blood electrolyte levels e) Monitoring intake and output

a) Maintaining NPO status for the first day or two b) Administering antiemetic agents d) Obtaining baseline blood electrolyte levels e) Monitoring intake and output Rationale: When hospitalization is necessary, oral food and fluids are withheld to allow the gut to rest. Antiemetic agents are ordered to help control nausea and vomiting. The woman is likely to be dehydrated, so the nurse would obtain baseline blood electrolyte levels and administer intravenous fluid and electrolyte replacement therapy as indicated. Once the nausea and vomiting subside, oral food and fluids are gradually reintroduced. Total parenteral nutrition or a feeding tube is used to prevent malnutrition only if the client does not improve with these interventions.

A pregnant client has been admitted with complaints of brownish vaginal bleeding. On examination there is an elevated hCG level, absent fetal heart sounds and a discrepancy between the uterine size and the gestational age. The nurse interprets these findings to suggest which of the following? a) Molar pregnancy b) Abruption of placenta c) Placenta previa d) Ectopic pregnancy

a) Molar pregnancy Rationale: The client is most likely experiencing molar pregnancy. In molar pregnancy, there is an abnormal proliferation and eventual degeneration of the trophoblastic villi. The signs and symptoms of molar pregnancy include brownish vaginal bleeding, elevated hCG levels, discrepancy between the uterine size and the gestational age, and absent fetal heart sounds. Abruption of placenta is characterized by premature separation of the placenta. Ectopic pregnancy is a condition where there is implantation of the blastocyst outside the uterus. In placenta previa the placental attachment is at the lower uterine segment.

A 24-year-old woman presents at the emergency room with abdominal pain, vaginal bleeding, and fatigue. What should the nurse's first action be? a) Monitor the patient for signs of shock. b) Ascertain from the patient if she has been sexually active to rule out pregnancy. c) Prepare for IV blood transfusion. d) Assume a pregnancy and prepare to determine the viability of the fetus.

a) Monitor the patient for signs of shock. Rationale: Anytime a patient presents with hemorrhage, the initial nursing consideration is assessment and evaluation of shock. This is also the case in the pregnant patient. If the mother is in shock the fetus will have lack of oxygen. Monitor the maternal status first. Ascertaining sexual activity to rule out pregnancy or assuming a pregnancy and preparing to determine the viability of the fetus is not the initial nursing consideration for this patient. The scenario does not indicate the need for a blood transfusion.

A 44-year-old client has lost several pregnancies over the last 10 years. For the past 3 months, she has had fatigue, nausea, and vomiting. She visits the clinic and takes a pregnancy test; the results are positive. Physical examination confirms a uterus enlarged to 13 weeks' gestation; fetal heart tones are heard. Ultrasound reveals that the client is experiencing some bleeding. Considering the client's prenatal history and age, what does the nurse recognize as the greatest risk for the client at this time? a) Pregnancy loss. b) Preterm labor. c) Premature birth. d) Hypertension.

a) Pregnancy loss. Rationale: The client's advanced maternal age (pregnancy in a woman 35 years or older) increases her risk for pregnancy loss. Hypertension, preterm labor, and prematurity are risks as this pregnancy continues. Her greatest risk at 13 weeks' gestation is losing this pregnancy.

A woman in week 35 of her pregnancy with severe hydramnios is admitted to the hospital. The nurse recognizes that which of the following is the biggest concern regarding this client? a) Preterm rupture of membranes followed by preterm birth b) Hemorrhaging c) Development of gestational trophoblastic disease d) Development of eclampsia

a) Preterm rupture of membranes followed by preterm birth Rationale: Even with precautions, in most instances of hydramnios, there will be preterm rupture of the membranes because of excessive pressure, followed by preterm birth. The other answers are not as big of concerns as preterm birth, in this situation.

A patient is admitted to labor and delivery for management of severe preeclampsia. An IV infusion of magnesium sulfate is started. What is the primary goal for magnesium sulfate therapy? a) Prevent maternal seizures b) Reverse edema c) Decrease blood pressure d) Decrease protein in urine

a) Prevent maternal seizures Rationale: The primary therapy goal for any preeclamptic patient is to prevent maternal seizures. Use of magnesium sulfate is the drug therapy of choice for severe preeclampsia and is only used to manage and attempt to prevent progression to eclampsia. Magnesium sulfate therapy does not have as its primary goal a decrease in blood pressure, a decrease in protein in the urine, nor the reversal of edema.

The following hourly assessments are obtained by the nurse on a patient with preeclampsia receiving Magnesium Sulfate: 97.3, P88, R10, blood pressure 148/110. What other priority physical assessment by the nurse should be implemented to assess for potential toxicity? a) Reflexes b) Magnesium sulfate level c) Lung sounds d) Oxygen saturation

a) Reflexes Rationale: Reflex assessment is part of the standard assessment for patients on magnesium sulfate. The first change when developing magnesium toxicity may be a decrease in reflex activity. The health care provider needs to be notified immediately. A change in lung sounds and oxygen saturation are not indicative of magnesium sulfate toxicity. Hourly blood draws to gain information on the magnesium sulfate level are not indicated.

Sometimes an ectopic pregnancy occurs outside the woman's uterus. This usually occurs in one of the fallopian tubes. If the embryo continues to grow, it may rupture the tube. What are the signs and symptoms of a ruptured fallopian tube? a) Shoulder pain b) Unilateral abdominal pain c) Rectal pain d) Bilateral abdominal pain

a) Shoulder pain Rationale: Rarely, a woman may present with late signs, such as shoulder pain or hypovolemic shock. These signs are associated with tubal rupture, which occurs when the pregnancy expands beyond the tube's ability to stretch.

A nurse is caring for a client who just experienced a miscarriage in her first trimester. When asked by the client why this happened, which is the best response from the nurse?

abnormal fetal development

A pregnant woman is admitted to the hospital with a diagnosis of placenta previa. Which action would be the priority for this woman on admission?

assessing fetal heart tones by use of an external monitor

A woman of 16 weeks' gestation telephones you because she has passed some "berry-like" blood clots and now has continued dark brown vaginal bleeding. Which of the following would you instruct the woman to do? a) "Come to the health care facility if uterine contractions begin." b) "Come to the health facility with any vaginal material passed." c) "Maintain bed rest and count the number of perineal pads used." d) "Continue normal activity, but take your pulse every hour."

b) "Come to the health facility with any vaginal material passed." Rationale: This is a typical time in pregnancy for gestational trophoblastic disease to present. Asking the woman to bring any material passed vaginally would be important so it can be assessed for this.

A client is 11 weeks pregnant after many years trying to conceive. After arriving home from a normal prenatal visit, she experiences mild cramping and has a gush of bright red vaginal bleeding. She calls the nurse and reports having soaked a pad with fresh blood in fewer than 30 minutes. The uterine cramping is worsening. What is the most appropriate response from the nurse? a) "This is nothing to worry about. Many women bleed during pregnancy." b) "You need to seek immediate attention from the primary care provider." c) "I am sorry. There is nothing you can do, because you are likely miscarrying." d) "Lie down and call your health care provider tomorrow if symptoms continue."

b) "You need to seek immediate attention from the primary care provider." Rationale: Pregnancy loss during the early weeks of pregnancy may seem like a heavy menstrual period. A primary care provider should assess blood loss of this amount with or without uterine cramping as soon as possible.

A client in her first trimester has just experienced a miscarriage. The nurse knows that which of the following is the most likely cause of the miscarriage? a) Lack of sufficient progesterone produced by the corpus luteum b) Abnormal fetal development c) Rejection of the embryo through an immune response d) Implantation abnormality

b) Abnormal fetal development Rationale: The most frequent cause of miscarriage in the first trimester of pregnancy is abnormal fetal development, due either to a teratogenic factor or to a chromosomal aberration. In other miscarriages, immunologic factors may be present or rejection of the embryo through an immune response may occur. Another common cause of early miscarriage involves implantation abnormalities. Miscarriage may also occur if the corpus luteum on the ovary fails to produce enough progesterone to maintain the decidua basalis.

Which of the following would be the physiologic basis for a placenta previa? a) A loose placental implantation b) A placenta with multiple lobes c) Low placental implantation d) A uterus with a midseptum

c) Low placental implantation Rationale: The cause of placenta previa is usually unknown, but for some reason the placenta is implanted low instead of high on the uterus.

A postpartum mother has the following lab data recorded: RH negative and rubella titer is positive. What is the appropriate nursing intervention? a) Assess the rubella of the baby b) Administer RhoGam within 72 hours. c) Administer rubella vaccine before discharge. d) Assess the RH of the baby.

b) Administer RhoGam within 72 hours. Rationale: The rubella is a virus and the mother has a positive titer indicating she is immune, this is important data for prenatal care and only has a bearing if the patient were negative. The RH is negative for the mother and the infant status is unknown, to protect future pregnancies the mother should be given RhoGam. It would not be appropriate to administer the rubella vaccine, assess the rubella titer of the baby, or assess the Rh status of the baby.

When providing counseling on early pregnancy loss, the nurse should include what as the most common cause for spontaneous abortion? a) Maternal smoking b) Chromosomal defect c) Lack of prenatal care d) The age of the mother

b) Chromosomal defect Rationale: The most common cause for the loss of a fetus in the first trimester is associated with a genetic defect or chromosomal defect. There is nothing that can be done and the mother should feel no fault. The nurse needs to educate the parents to speak with a health care provider for further information and questions related to genetic testing. Early pregnancy loss is not associated with maternal smoking, lack of prenatal care, or the age of the mother.

A pregnant client with multiple gestation arrives at the maternity clinic for a regular antenatal check up. The nurse would be aware of the client's risk of perinatal complications including which of the following? a) Maternal hypotension b) Congenital anomalies c) Post-term birth d) Fetal non-immune hydrops

b) Congenital anomalies Rationale: Multiple gestation involves two or more fetuses. The perinatal complications associated with multiple pregnancy include preterm birth, maternal hypertension and congenital anomalies. Fetal non-immune hydrops occurs in the infection of pregnant clients with parvovirus. Post-term birth, maternal hypotension and fetal non-immune hydrops are not seen as complications of multiple pregnancy.

Which of the following nursing diagnoses would be most appropriate for a woman diagnosed with pregnancy-induced hypertension? a) Risk for injury related to fetal distress b) Deficient fluid volume related to vasospasm of arteries c) Imbalanced nutrition related to decreased sodium levels d) Ineffective tissue perfusion related to poor heart contraction

b) Deficient fluid volume related to vasospasm of arteries Rationale: Hypertension of pregnancy is caused by vascular spasm. This leads to increased blood pressure and edema. Extensive edema leads to a deficiency of fluid volume.

A client visits a health care facility with complaints of amenorrhea for 10 weeks, fatigue, and breast tenderness. Which of the following additional signs and symptoms suggest the presence of molar pregnancy? Select all that apply. a) Whitish discharge from the vagina b) Elevated hCG levels c) Hyperemesis gravidarum d) Dyspareunia e) Absence of fetal heart sound

b) Elevated hCG levels c) Hyperemesis gravidarum e) Absence of fetal heart sound Rationale: The signs and symptoms of molar pregnancy include an elevated hCG level, absence of fetal heart sounds, and hyperemesis gravidarum. Whitish discharge from the vagina and dyspareunia (painful sexual intercourse) are seen in cases of infection, and not in molar pregnancy. In molar pregnancy a brownish vaginal bleeding is seen.

A novice nurse asks to be assigned to the least complex antepartum patient. Which of the following conditions would necessitate the least complex care requirements? a) Pre-ecalmpsia. b) Gestational hypertension. c) Placenta previa. d) Abruptio placenta.

b) Gestational hypertension. Rationale: Hypertensive disorders represent the most common complication of pregnancy. Gestational hypertension is elevated blood pressure without proteinuria, other signs of pre-eclampsia, or pre-existing hypertension. Abruptio placenta (separation of the placenta from the uterine wall), placenta previa (placenta covering the cervical os), and pre-eclampsia are high-risk, potentially life-threatening conditions for the fetus and mother during labor and birth.

To which of the following patients being discharged for home must the nurse stress that it is absolutely critical the patient return for monthly follow-up visits? A woman who: a) Has experienced a complete spontaneous abortion b) Has experienced a molar pregnancy c) Has experienced an ectopic pregnancy d) Is Rh negative

b) Has experienced a molar pregnancy Rationale: Molar pregnancies can indicate the possibility of developing malignancy. The woman will need close observation and follow-up for the year following the diagnosis. Follow-up visits after an ectopic pregnancy or a complete spontaneous abortion are typically scheduled at six weeks, not monthly. A woman who is Rh negative does not need a follow-up visit because of her Rh status.

A pregnant client with severe pre-eclampsia has developed the HELLP syndrome. In addition to the observations necessary for pre-eclampsia, what other nursing intervention is critical for this patient? a) Administration of a tocolytic, if prescribed b) Observation for bleeding c) Monitoring for infection d) Maintaining a patent airway

b) Observation for bleeding Rationale: Because of the low platelet count associated with this condition, women with the HELLP syndrome need extremely close observation for bleeding, in addition to the observations necessary for pre-eclampsia. Maintaining a patent airway is a critical intervention needed for a patient with eclampsia while she is having a seizure. Administration of a tocolytic would be appropriate for halting labor. Monitoring for infection is not a priority intervention in this situation.

A pregnant woman with preeclampsia is to receive magnesium sulfate IV. Which of the following assessments would be most important prior to administering a new dose? a) Anxiety level b) Patellar reflex c) Blood pressure d) Pulse rate

b) Patellar reflex Rationale: A symptom of magnesium sulfate toxicity is loss of deep tendon reflexes. Assessing for one of these before administration is assurance the drug administration will be safe.

A woman in week 35 of her pregnancy with severe hydramnios is admitted to the hospital. The nurse recognizes that which of the following is the biggest concern regarding this client? a) Development of gestational trophoblastic disease b) Preterm rupture of membranes followed by preterm birth c) Development of eclampsia d) Hemorrhaging

b) Preterm rupture of membranes followed by preterm birth Rationale: Even with precautions, in most instances of hydramnios, there will be preterm rupture of the membranes because of excessive pressure, followed by preterm birth. The other answers are not as big of concerns as preterm birth, in this situation.

A client in her 38th week of gestation is admitted into the labor and birth unit with painless bleeding from the vagina. The client is diagnosed with placenta previa. When reviewing the client's history, which of the following would the nurse identify as a risk factor for placenta previa? a) Maternal age more than 30 years b) Previous cesarean birth c) Primigravida status d) Living in coastal areas

b) Previous cesarean birth Rationale: A previous cesarean birth is a risk factor for developing placenta previa. This is due to the damage caused to the endometrial tissue. Multiparity, and not a primigravida status, predisposes to placenta previa. Maternal age over 35 years, and not just more than 30 years, is considered another risk factor. Placenta previa is more common among those living in high altitudes, and not among those living in coastal areas.

What makes the diagnosis of gestational hypertension different from the diagnosis of preeclampsia? a) Ketonuria b) Proteinuria c) Severity of hypertension d) The hypertension of gestation disappears after delivery. The hypertension of preeclampsia does not.

b) Proteinuria Rationale: Gestational hypertension is the current term used to describe elevated blood pressure (greater than or equal to 140/90 mm/Hg) that develops for the first time during pregnancy without the presence of protein in the urine.

You are caring for a patient with preeclampsia. You know that you need to auscultate this patient's lung sounds every two hours. Why would you do this? a) Pulmonary emboli b) Pulmonary edema c) Pulmonary atelectasis d) Pulmonary hypertension

b) Pulmonary edema Rationale: In the hospital, monitor blood pressure at least every four hours for mild preeclampsia and more frequently for severe disease. In addition, it is important to auscultate the lungs every two hours. Adventitious sounds may indicate, developing pulmonary edema.

A patient with preeclampsia is receiving magnesium sulfate. Which of the following nursing assessments should be ongoing while the medication is being administered? a) Hemoglobin. b) Respiratory rate. c) Urine protein. d) Ability to sleep.

b) Respiratory rate. Rationale: The level of magnesium in therapeutic range is 4 to 8 mg/dL. If magnesium toxicity occurs, one sign in the patient will be a decrease in the respiratory rate and a potential respiratory arrest. Respiratory rate will be monitored when on this medication. The patient's hemoglobin and ability to sleep are not factors for on-going assessments for the patient on magnesium sulfate. Urinary output is measured hourly on the preeclamptic patient receiving magnesium sulfate, but urine protein is not an ongoing assessment.

A pregnant woman at 12 weeks' gestation calls you because she has begun minimal fresh vaginal spotting. She is distressed because her physician says she is not going to do anything for her but "wait and see." Which of the following would you suggest? a) Suggest she take an over-the-counter tocolytic just to feel secure. b) Tell her that medication to prolong a 12-week pregnancy usually is not advised. c) Advise her to ask for a second physician opinion. d) Explain that her doctor meant for her to maintain strict bed rest by "wait and see."

b) Tell her that medication to prolong a 12-week pregnancy usually is not advised. Rationale: Because many early pregnancy losses occur as the result of chromosome abnormalities, an aggressive approach to prolong these is not usually recommended.

A pregnant woman at 12 weeks' gestation calls you because she has begun minimal fresh vaginal spotting. She is distressed because her physician says she is not going to do anything for her but "wait and see." Which of the following would you suggest? a) Suggest she take an over-the-counter tocolytic just to feel secure. b) Tell her that medication to prolong a 12-week pregnancy usually is not advised. c) Explain that her doctor meant for her to maintain strict bed rest by "wait and see." d) Advise her to ask for a second physician opinion.

b) Tell her that medication to prolong a 12-week pregnancy usually is not advised. Rationale: Because many early pregnancy losses occur as the result of chromosome abnormalities, an aggressive approach to prolong these is not usually recommended.

A woman and her partner present at her first antenatal obstetrics appointment. She is 6 weeks pregnant. Her blood tests show that she is Rh negative although she has not previously known this. She has no other children but suspects that she miscarried early in a pregnancy 2 years previously. She felt fine afterward so she never received medical attention for that suspected episode. Her partner reports that he is also Rh negative. What action will the nurse be likely to take? a) If an antibody screen is positive, the nurse will administer RhoGam to the patient. b) The nurse will perform all normal procedures and follow-up tasks because the Rh status of the patient and the child are not a concern at this time. c) The nurse will arrange for an amniocentesis to assess hemolytic disease in the fetus. d) If an antibody screen is negative, the nurse will administer RhoGam to the patient.

b) The nurse will perform all normal procedures and follow-up tasks because the Rh status of the patient and the child are not a concern at this time. Rationale: If the woman is Rh negative and her partner is Rh negative the fetus will also be Rh negative and the woman will not require treatment with RhoGam. Therefore options A, B, and C are incorrect.

A woman you care for has an Rh-negative blood type. Following the birth of her infant, you administer her RhIG (D immune globulin). The purpose of this is to a) promote maternal D antibody formation. b) prevent maternal D antibody formation. c) stimulate maternal D immune antigens. d) prevent fetal Rh blood formation.

b) prevent maternal D antibody formation. Rationale: Because RhIG contains passive antibodies, the solution will prevent the woman from forming long-lasting antibodies.

A client is admitted at 22 weeks' gestation with advanced cervical dilatation to 5 centimeters, cervical insufficiency, and a visible amniotic sac at the cervical opening. What is the primary goal for this client at this point?

bed rest to maintain pregnancy as long as possible At 22 weeks' gestation, the fetus is not viable. The woman would be placed on bed rest, total, with every attempt made to halt any further progression of dilatation as long as possible. The nurse would not want this fetus to be born vaginally at this stage of gestation. It is not the nurse's responsibility to notify the client's social support of a possible loss of the pregnancy. It is not appropriate at this time to educate the mother on causes of cervical insufficiency for future pregnancies

Select the statement by the pregnant woman that indicates the need for more teaching about preeclampsia. a) "I will count my baby's movements twice a day." b) "If I have a slight headache I'll take Tylenol and call if unrelieved." c) "If I have changes in my vision, I will lie down and rest." d) "I will weigh myself every morning after voiding before breakfast."

c) "If I have changes in my vision, I will lie down and rest." Rationale: Changes in the visual field may indicate the patient has moved from preeclampsia to severe preeclampisa and is at risk for developing a seizure due to changes in cerebral blood flow. The patient would require immediate assessment and intervention. Options B, C, and D would not indicate that more teaching about preeclampsia is indicated.

A woman who is 31 weeks pregnant presents at the emergency room with bright red vaginal bleeding. She says the onset of the bleeding was sudden and she has no pain. The nurse is most likely to assist the physician or technician with which exam? a) A blood transfusion b) An abdominal ultrasound c) A transvaginal ultrasound d) A digital cervical exam

c) A transvaginal ultrasound Rationale: The use of a transvaginal ultrasound is the diagnostic test of choice; it is 100% accurate in prediction of placenta previa, while abdominal ultrasound is only 95% accurate. A digital cervical exam is contraindicated in this patient and the scenario described does not indicate the need for a blood transfusion.

The nurse is caring for a woman who is 28 weeks pregnant and has been hospitalized with moderate bleeding which seems to be stabilizing; she is asleep on her side. While checking the fetal monitor tape the nurse notices that the baseline has gradually increased with late decelerations. What is the first action the nurse will take? a) Reposition the mother in a semirecumbent position on her back. b) Notify the physician. c) Administer oxygen to the mother. d) Decrease the rate of IV fluids.

c) Administer oxygen to the mother. Rationale: The fetus is showing signs of fetal distress. The immediate treatment is using oxygen, putting the patient in a side lying position, and increasing IV fluids. The interventions are to improve fetal circulation. Therefore options B, C, and D are incorrect answers.

A pregnant woman is admitted to the hospital with a diagnosis of placenta previa. Which of the following would be the priority for this woman on admission? a) Performing a vaginal examination to assess the extent of bleeding b) Helping the woman remain ambulatory to reduce bleeding c) Assessing fetal heart tones by use of an external monitor d) Assessing uterine contractions by an internal pressure gauge

c) Assessing fetal heart tones by use of an external monitor Rationale: Not disrupting the placenta is a prime responsibility. An internal monitor, a vaginal examination, and remaining ambulatory could all do this and thus are contraindicated.

Which of the following would the nurse interpret as indicating that a pregnant client with gestational hypertension has developed severe preeclampsia? a) Proteinuria of 300 mg/24 hours b) Blood pressure of 150/100 mm Hg c) Blurred vision d) Mild facial edema

c) Blurred vision Rationale: Visual symptoms such as blurred vision and blind spots suggest severe preeclampsia. Severe preeclampsia is characterized by a blood pressure of 160/110 mm Hg. Mild facial edema or hand edema occurs with mild preeclampsia. Proteinuria in severe preeclampsia is greater than 500 mg/24 hours.

A client has been admitted to the hospital with a diagnosis of severe pre-eclampsia. Which of the following is the priority nursing? a) Check for vaginal bleeding every 15 minutes b) Keep the client on her side so that secretions can drain from her mouth c) Confine the client to bed rest in a darkened room d) Administer oxygen by face mask

c) Confine the client to bed rest in a darkened room Rationale: With severe pre-eclampsia, most women are hospitalized so that bed rest can be enforced and a woman can be observed more closely than she can be on home care. Darken the room if possible because a bright light can also trigger seizures. The other interventions listed pertain to a client who has experienced a seizure and has thus progressed to eclampsia.

When administering magnesium sulfate to a woman with severe preeclampsia, which finding would alert the nurse to the development of magnesium toxicity? a) Seizures b) Elevated liver enzymes c) Diminished reflexes d) Serum magnesium level of 6.5 mEq/L

c) Diminished reflexes Rationale: Diminished or absent reflexes occur when a client develops magnesium toxicity. Elevated liver enzymes are unrelated to magnesium toxicity and may indicate the development of HELLP syndrome. The onset of seizure activity indicates eclampsia. A serum magnesium level of 6.5 mEq/L would fall within the therapeutic range of 4 to 7 mEq/L.

A pregnant woman is diagnosed with abruptio placentae. When reviewing the woman's medical record, which of the following would the nurse expect to find? a) Bright red vaginal bleeding b) Fetal heart rate within normal range c) Firm, rigid uterus on palpation d) Absence of pain

c) Firm, rigid uterus on palpation Rationale: The uterus is firm to rigid to the touch with abruptio placentae; it is soft and relaxed with placenta previa. Bleeding associated with abruptio placentae occurs suddenly and is usually dark in color. Bleeding also may not be visible. Bright red vaginal bleeding is associated with placenta previa. Fetal distress or absent fetal heart rate may be noted with abruptio placentae. The woman with abruptio placentae usually experiences constant uterine tenderness on palpation.

A woman is admitted with a diagnosis of ectopic pregnancy. For which of the following would you anticipate beginning preparation? a) Bed rest for the next 4 weeks. b) Intravenous administration of a tocolytic. c) Immediate surgery. d) Internal uterine monitoring.

c) Immediate surgery. Rationale: Ectopic pregnancy means an embryo has implanted outside the uterus, usually in the fallopian tube. Surgery is usually necessary to remove the growing structure before the tube ruptures or repair the tube if rupture has already occurred. Bed rest will not correct the problem of an ectopic pregnancy. Administering a tocolytic is not indicated, nor is internal uterine monitoring. This makes options A, B, and D incorrect

Which of the following would the nurse include in the plan of care for a woman with premature rupture of membranes if her fetus's lungs are mature? a) Administration of corticosteroids b) Observation for signs of infection c) Labor induction d) Reduction in physical activity level

c) Labor induction Rationale: With premature rupture of membranes in a woman whose fetus has mature lungs, induction of labor is initiated. Reducing physical activity, observing for signs of infection, and giving corticosteroids may be used for the woman with PROM when the fetal lungs are immature.

Which of the following would the nurse prepare to administer if ordered as treatment for an unruptured ectopic pregnancy? a) Promethazine b) Oxytocin c) Methotrexate d) Ondansetron

c) Methotrexate Rationale: Methotrexate, a folic acid antagonist that inhibits cell division in the developing embryo, is most commonly used to treat ectopic pregnancy. Oxytocin is used to stimulate uterine contractions and would be inappropriate for use with an ectopic pregnancy. Promethazine and ondansetron are antiemetics that may be used to treat hyperemesis gravidarum.

A woman is 9 weeks gestation and admitted to the obstetrical unit for hyperemesis gravidarum. The highest priority intervention the nurse should anticipate is which of the following? a) Administration of antiemetics b) IV rehydration c) NPO for 24 hours d) Bed rest with bathroom privileges

c) NPO for 24 hours Rationale: The initial intervention to have priority is to stop all food and fluids by mouth until the vomiting has stopped. The next steps will depend on the severity of the hyperemesis gravidarum. Use of medications should be based on the severity of the hyperemesis gravidarum and only under orders as most medications are pregnancy category C. IV rehydration may be ordered by the physician.

A woman in labor has sharp fundal pain accompanied by slight vaginal bleeding. Which of the following would be the most likely cause of these symptoms? a) Possible fetal death or injury. b) Placenta previa obstructing the cervix. c) Premature separation of the placenta. d) Preterm labor that was undiagnosed.

c) Premature separation of the placenta. Rationale: Premature separation of the placenta begins with sharp fundal pain, usually followed by vaginal bleeding. Placenta previa usually produces painless bleeding; Preterm labor contractions are more often described as cramping. Possible fetal death or injury does not present with sharp fundal pain. It is usually painless.

When assessing a woman with an ectopic pregnancy, which of the following would lead the nurse to suspect that the tube has ruptured? a) Breast tenderness b) Vaginal spotting c) Referred shoulder pain d) Nausea

c) Referred shoulder pain Rationale: Referred pain to the shoulder area indicates bleeding into the abdomen caused by phrenic nerve irritation when a tubal pregnancy ruptures. Vaginal spotting, nausea, and breast tenderness are typical findings of early pregnancy and an unruptured ectopic pregnancy.

Which measure would be most effective in preventing isoimmunization during pregnancy? a) Cerclage b) Blood typing of mothers with type A or B blood c) RhoGAM administration to Rh-negative women d) Amniocentesis

c) RhoGAM administration to Rh-negative women Rationale: Rh incompatibility can be prevented with the use of RhoGAM. Hemolysis associated with ABO incompatibility is limited to mothers with type O blood and their fetuses with type A or B blood. Amniocentesis would be appropriate for treatment of polyhydramnios, not isoimmunization. Cerclage is a treatment for cervical insufficiency.

A 45-year-old pregnant woman with type O blood has had an amniocentesis to rule out Down syndrome. The fetus has type AB blood. What can the nurse warn the patient is a likely outcome if some fetal blood mixed with maternal blood during the procedure? a) The baby will develop hemolytic anemia. b) Preterm delivery c) The baby will have postdelivery jaundice. d) Placental abruption

c) The baby will have postdelivery jaundice. Rationale: The infant and mother have ABO incompatibility. The result is a development of antibodies and breaking down of the blood resulting in jaundice in the infant after delivery. The mixing of some fetal blood with maternal blood during the amniocentesis would not cause placental abruption or preterm delivery. Hemolytic anemia is caused by Rh incompatibility, not ABO incompatibility.

A woman in week 16 of her pregnancy calls her physician's office to report that she has experienced abdominal cramping, cervical dilation, vaginal spotting, and the passing of tissue. The nurse instructs the client to bring the passed tissue to the hospital with her. What is the correct rationale for this instruction? a) To determine the stage of development of the fetus b) To determine whether infection is present c) To determine whether gestational trophoblastic disease is present d) To determine whether the fetus is viable

c) To determine whether gestational trophoblastic disease is present Rationale: Gestational trophoblastic disease is abnormal proliferation and then degeneration of the trophoblastic villi. The embryo fails to develop beyond a primitive start. Abnormal trophoblast cells must be identified because they are associated with choriocarcinoma, a rapidly metastasizing malignancy. This is why it is important for any woman who begins to miscarry at home to bring any clots or tissue passed to the hospital with her. The presence of clear fluid-filled cysts changes the diagnosis from a simple miscarriage to gestational trophoblastic disease. The client is not instructed to bring in passed tissue to determine whether infection is present or the fetus is viable or to determine the stage of development of the fetus.

A woman at 8 weeks' gestation is admitted for ectopic pregnancy. She is asking why this has occurred. The nurse knows that which of the following is a known risk factor for ectopic pregnancy? a) High number of pregnancies b) Multiple gestation pregnancy c) Use of IUD for contraception d) Use of oral contraceptives

c) Use of IUD for contraception Rationale: Use of an IUD with progesterone has a known increased risk for development of ectopic pregnancies. The nurse needs to complete a full history of the patient to determine if she had any other risk factors for an ectopic pregnancy. Adhesions, scarring, and narrowing of the tubal lumen may block the zygote's progress to the uterus. Any condition or surgical procedure that can injure a fallopian tube increases the risk. Examples include salpingitis, infection of the fallopian tube, endometriosis, history of prior ectopic pregnancy, any type of tubal surgery, congenital malformation of the tube, and multiple elective abortions. Conditions that inhibit peristalsis of the tube can result in tubal pregnancy. Hormonal factors may play a role because tubal pregnancy occurs more frequently in women who take fertility drugs or who use progesterone intrauterine contraceptive devices (IUDs). A high number of pregnancies, multiple gestation pregnancy, and the use of oral contraceptives are not known risk factors for ectopic pregnancy.

When providing counseling on early pregnancy loss, the nurse should discuss which factor as the most common cause for spontaneous abortion?

chromosomal abnormality The most common cause for the loss of a fetus in the first trimester is associated with a genetic defect or chromosomal abnormality. There is nothing that can be done, and the mother should feel no fault. The nurse needs to educate the parents to speak with a health care provider for further information and questions related to genetic testing. Early pregnancy loss is not associated with maternal smoking, lack of prenatal care, or the age of the mother.

Which of the following is the best question the nurse can ask a woman who is leaving the hospital after experiencing a complete spontaneous abortion? a) "Did you know that 75 percent of women who are trying to get pregnant experience spontaneous abortions like you have?" b) "Are you going to try again?" c) "May I give you some resources that you can use to try to stop smoking?" d) "Do you have someone to talk to or may I give you the names and numbers for some possible grief counselors?"

d) "Do you have someone to talk to or may I give you the names and numbers for some possible grief counselors?" Rationale: When a woman has a spontaneous abortion one important consideration is the emotional needs of the woman once she is home. She may not want to talk about the loss for a period of time, but the nurse needs to determine her support system for the future. Asking the woman if she is "going to try again" is an inappropriate question for the nurse to ask, and diminishes the experience of having a spontaneous abortion. Giving the woman statistical information on spontaneous abortions is not appropriate when this patient needs support and caring concern. Offering to give the patient resources to aid in smoking cessation is not addressed in the scenario so this is an inappropriate response.

A pregnant client in her 22nd week of gestation arrives at the healthcare facility with complaints of excessive vaginal bleeding and absence of fetal movements. She is diagnosed as having second trimester fetal loss. Which of the following would the nurse anticipate as the cause of second trimester fetal loss? a) Ectopic pregnancy b) Congenital malformations c) Placenta previa d) Cervical incompetence

d) Cervical incompetence Rationale: The nurse should identify cervical incompetence as the cause for second trimester fetal loss. Cervical incompetence is a condition where there is painless cervical dilatation and results in second trimester fetal loss or can progress to preterm premature rupture of membranes. Ectopic pregnancy, congenital malformations and placenta previa are not involved in causing second trimester fetal loss. Ectopic pregnancy usually leads to first trimester fetal loss. Placenta previa is a condition in which there is implantation of the placenta to the lower uterine segment. Congenital malformations result in first trimester fetal loss.

A 28-year-old woman presents in the emergency room with severe abdominal pain. She has not had a normal period for 2 months but she reports that that is not abnormal for her. She has a history of endometriosis. What might the nurse suggest to the physician as a possible cause of the patient's abdominal pain? a) Healthy pregnancy b) Placenta previa c) Molar pregnancy d) Ectopic pregnancy

d) Ectopic pregnancy Rationale: Ectopic pregnancy can present with severe unilateral abdominal pain. Given the history of the client , the amount of pain, the possibility of ectopic pregnancy needs to be considered. A healthy pregnancy would not present with severe abdominal pain unless the patient were term and she was in labor. With a molar pregnancy the woman typically presents between 8 to 16 weeks' gestation with complaints of painless (usually) brown to bright red vaginal bleeding. Placenta previa typically presents with painless, bright red bleeding that begins with no warning.

A woman is being admitted to your hospital unit for severe preeclampsia. When deciding on where to place her, which of the following areas would be most appropriate? a) Near the elevator so she can be transported quickly b) Near the nurse's station so she can be observed closely c) By the nursery so she can maintain hope she will have a child d) In the back hallway where there is a quiet, private room

d) In the back hallway where there is a quiet, private room Rationale: A sudden noise can trigger a seizure in a severely preeclamptic woman. Room placement, therefore, should not be near noise, such as the nursery, the elevator, or nurse's station.

During a routine prenatal visit, a client is found to have proteinuria and a blood pressure rise to 140/90 mm Hg. The nurse recognizes that the client has which of the following conditions? a) Eclampsia b) Severe pre-eclampsia c) Gestational hypertension d) Mild pre-eclampsia

d) Mild pre-eclampsia Rationale: A woman is said to have gestational hypertension when she develops an elevated blood pressure (140/90 mm Hg) but has no proteinuria or edema. If a seizure from gestational hypertension occurs, a woman has eclampsia, but any status above gestational hypertension and below a point of seizures is pre-eclampsia. A woman is said to be mildly pre-eclamptic when she has proteinuria and a blood pressure rise to 140/90 mm Hg, taken on two occasions at least 6 hours apart. A woman has passed from mild to severe pre-eclampsia when her blood pressure rises to 160 mm Hg systolic and 110 mm Hg diastolic or above on at least two occasions 6 hours apart at bed rest (the position in which blood pressure is lowest) or her diastolic pressure is 30 mm Hg above her prepregnancy level. Marked proteinuria, 3+ or 4+ on a random urine sample or more than 5 g in a 24-hour sample, and extensive edema are also present. A woman has passed into eclampsia when cerebral edema is so acute a grand-mal seizure (tonic-clonic) or coma has occurred.

When assessing a pregnant woman with vaginal bleeding, which finding would lead the nurse to suspect an inevitable abortion? a) No passage of fetal tissue b) Slight vaginal bleeding c) Closed cervical os d) Strong abdominal cramping

d) Strong abdominal cramping Rationale: Strong abdominal cramping is associated with an inevitable abortion. Slight vaginal bleeding early in pregnancy and a closed cervical os are associated with a threatened abortion. With an inevitable abortion, passage of the products of conception may occur. No fetal tissue is passed with a threatened abortion.

In returning to the hospital floor after a weekend off, the nurse takes over care of a pregnant patient who is resting in a darkened room. The patient is receiving betamethasone and magnesium sulfate. What could the nurse deduce from those findings? a) The patient is suffering from eclampsia and the care team is attempting to prevent stroke and induce labor. b) The patient is suffering from mild preeclampsia and the care team is attempting to stabilize her and the baby before discharging her to home. c) The patient is suffering from hypertension and the care team is trying to lower her blood pressure so that she may return home until the baby is full term. d) The patient is suffering from severe preeclampsia and the care team is attempting to prevent advancement of the disorder to eclampsia; they are attempting to help the baby's lungs mature quickly so that they can deliver as soon as possible.

d) The patient is suffering from severe preeclampsia and the care team is attempting to prevent advancement of the disorder to eclampsia; they are attempting to help the baby's lungs mature quickly so that they can deliver as soon as possible. Rationale: The administration of magnesium sulfate is to relax the skeletal muscles and raise the threshold for a seizure. The administration of the betamethasone is to try and hasten the maturity of the fetus' lungs for delivery. This woman is in advanced preeclampsia and must be monitored for progression to eclampsia. The scenario described does not indicate a patient with hypertension who may be discharged home once the condition is under control. A woman in eclampsia would either be seizing or comatose, not resting in a quiet room. Symptoms of mild preeclampsia are limited to slightly elevated blood pressure and small amounts of protein in the urine. Betamethasone may be indicated at this time.

The nurse through assessment can best differentiate between placenta previa and abruptio placentae by which of the following signs and or symptoms. a) Bleeding amount and consistency. b) Low back pain. c) Shape of the abdomen. d) Uterine tone and contractions of the uterus.

d) Uterine tone and contractions of the uterus. Rationale: With placenta previa the bleeding is often bright red and painless, with abruptio placentae the bleeding is usually dark and painful. The uterus is firm and hard and painful with the abruption; the uterus is often soft and lacks tone with the previa. The contractions of the uterus, low back pain, or the shape of the abdomen do not help to distinguish between placenta previa and abruption placentae.

A nurse is assessing a pregnant client for the possibility of preexisting conditions that could lead to complications during pregnancy. The nurse suspects that the woman is at risk for hydramnios based on which preexisting condition?

diabetes Approximately 18% of all women with diabetes will develop hydramnios during their pregnancy. Hydramnios occurs in approximately 3% of all pregnancies and is associated with fetal anomalies of development.

A woman with severe preeclampsia is receiving magnesium sulfate. The woman serum magnesium level is 9.0mEq/L. Which finding would the nurse most likely note?

diminished reflexes Diminished or absent reflexes occur when a client develops magnesium toxicity, serum levels greater than 8.0 mEq/L. Elevated liver enzymes are unrelated to magnesium toxicity and may indicate the development of HELLP syndrome. The onset of seizure activity indicates eclampsia. A serum magnesium level of 6.5 mEq/L would fall within the therapeutic range of 4 to 7 mEq/L.

A woman is being closely monitored and treated for severe preeclampsia with magnesium sulfate. Which finding would alert the nurse to the development of magnesium toxicity in this client?

diminished reflexes Diminished or absent reflexes occur when a client develops magnesium toxicity. Elevated liver enzymes are unrelated to magnesium toxicity and may indicate the development of HELLP syndrome. The onset of seizure activity indicates eclampsia. A serum magnesium level of 6.5 mEq/L would fall within the therapeutic range of 4 to 7 mEq/L.

A 28-year-old woman presents in the emergency department with severe abdominal pain. She has not had a normal period for 2 months, but she reports that that is not abnormal for her. She has a history of endometriosis. What might the nurse suggest to the primary care provider as a possible cause of the client's abdominal pain?

ectopic pregnancy Ectopic pregnancy can present with severe unilateral abdominal pain. Given the history of the client and the amount of pain, the possibility of ectopic pregnancy needs to be considered. A healthy pregnancy would not present with severe abdominal pain unless the client were term and she was in labor. With a molar pregnancy the woman typically presents between 8 to 16 weeks' gestation reporting painless (usually) brown to bright red vaginal bleeding. Placenta previa typically presents with painless, bright red bleeding that begins with no warning.

A nurse is explaining to a group of nursing students that eclampsia or seizures in pregnant women are preceded by an acute increase in maternal blood pressure. What are features of an acute increase in blood pressure? Select all that apply.

proteinuria hypereflexia blurring of vision Eclampsia is usually preceded by an acute increase in blood pressure as well as worsening signs of multi-organ system failure seen as increasing liver enzymes, proteinuria, and symptoms such as blurred vision and hyperreflexia. Hyperglycemia and auditory hallucinations are not seen with an acute increase in maternal blood pressure.

A nurse suspects that a client is developing HELLP syndrome. The nurse notifies the health care provider based on which finding?

elevated liver enzymes

A woman with an incomplete abortion is to receive misoprostol. The woman asks the nurse, "Why am I getting this drug?" The nurse responds to the client, integrating understanding that this drug achieves which effect?

ensures passage of all the products of conception

A nurse is monitoring a client with PROM who is in labor and observes meconium in the amniotic fluid. What does the observation of meconium indicate?

fetal distress related to hypoxia When meconium is present in the amniotic fluid, it typically indicates fetal distress related to hypoxia. Meconium stains the fluid yellow to greenish brown, depending on the amount present. A decreased amount of amniotic fluid reduces the cushioning effect, thereby making cord compression a possibility. A foul odor of amniotic fluid indicates infection. Meconium in the amniotic fluid does not indicate CNS involvement.

A woman at 10 weeks gestation comes to the clinic for an evaluation. Which assessment finding should the nurse prioritize?

fundal height measurement of 18 cm

A client has come to the office for a prenatal visit during her 22nd week of gestation. On examination, it is noted that her blood pressure has increased to 138/90 mm Hg. Her urine is negative for proteinuria. The nurse recognizes which factor as the potential cause?

gestational hypertension

A client has come to the office for a prenatal visit during her 22nd week of gestation. On examination, it is noted that her blood pressure has increased to 138/90 mm Hg. Her urine is negative for proteinuria. The nurse recognizes which factor as the potential cause?

gestational hypertension Gestational hypertension is characterized by hypertension without proteinuria after 20 weeks of gestation resolving by 12 weeks postpartum. It is defined as systolic blood pressure of greater than 140 mm Hg and/or diastolic of greater than 90 mm Hg on at least two occasions at least 6 hours apart after the 20th week of gestation, in women known to be normotensive prior to this time and prior to pregnancy. HELLP is an acronym that refers to hemolysis, elevated liver enzymes, and low platelets. Preeclampsia may result if hypertension is not controlled or advances to a more severe state.

A novice nurse asks to be assigned to the least complex antepartum client. Which condition would necessitate the least complex care requirements?

gestational hypertension Hypertensive disorders represent the most common complication of pregnancy. Gestational hypertension is elevated blood pressure without proteinuria, other signs of preeclampsia, or preexisting hypertension. Abruptio placenta (separation of the placenta from the uterine wall), placenta previa (placenta covering the cervical os), and preeclampsia are high-risk, potentially life-threatening conditions for the fetus and mother during labor and birth.

A nurse is taking a history of a client at 5 weeks' gestation in the prenatal clinic; however, the client is reporting dark brown vaginal discharge, nausea, and vomiting. Which diagnosis should the nurse suspect?

gestational trophoblastic disease

A pregnant client has been admitted with reports of brownish vaginal bleeding. On examination there is an elevated hCG level, absent fetal heart sounds, and a discrepancy between the uterine size and the gestational age. The nurse interprets these findings to suggest which condition?

gestational trophoblastic disease

A nurse is assessing a client diagnosed with mild preeclampsia. The nurse suspects that the client has developed severe preeclampsia based on which finding?

urine output of less than 400 mL/24 hours Severe preeclampsia may develop suddenly and bring with it high blood pressure of more than 160/110 mm Hg, proteinuria of more than 5 g in 24 hours, oliguria of less than 400 mL in 24 hours, cerebral and visual symptoms, and rapid weight gain. Mild facial edema or hand edema occurs with mild preeclampsia. Proteinuria in severe preeclampsia is greater than 500 mg/24 hours.

A pregnant client has been admitted with reports of brownish vaginal bleeding. On examination there is an elevated hCG level, absent fetal heart sounds, and a discrepancy between the uterine size and the gestational age. The nurse interprets these findings to suggest which condition?

gestational trophoblastic disease The client is most likely experiencing gestational trophoblastic disease or a molar pregnancy. In gestational trophoblastic disease or molar pregnancy, there is an abnormal proliferation and eventual degeneration of the trophoblastic villi. The signs and symptoms of molar pregnancy include brownish vaginal bleeding, elevated hCG levels, discrepancy between the uterine size and the gestational age, and absent fetal heart sounds. Abruption of placenta is characterized by premature separation of the placenta. Ectopic pregnancy is a condition where there is implantation of the blastocyst outside the uterus. In placenta previa the placental attachment is at the lower uterine segment.

A client has been admitted with abruptio placentae. She has lost 1,200 mL of blood, is normotensive, and ultrasound indicates approximately 30% separation. The nurse documents this as which classification of abruptio placentae?

grade 2 The classifications for abruptio placentae are: grade 1 (mild) - minimal bleeding (less than 500 mL), 10% to 20% separation, tender uterus, no coagulopathy, signs of shock or fetal distress; grade 2 (moderate) - moderate bleeding (1,000 to 1,500 mL), 20% to 50% separation, continuous abdominal pain, mild shock, normal maternal blood pressure, maternal tachycardia; grade 3 (severe) - absent to moderate bleeding (more than 1,500 mL), more than 50% separation, profound shock, dark vaginal bleeding, agonizing abdominal pain, decreased blood pressure, significant tachycardia, and development of disseminated intravascular coagulopathy. There is no grade 4

A nurse is providing care to a client who has been diagnosed with a common benign form of gestational trophoblastic disease. The nurse identifies this as:

hydatidiform mole. Gestational trophoblastic disease comprises a spectrum of neoplastic disorders that originate in the placenta. The two most common types are hydatidiform mole (partial or complete) and choriocarcinoma. Ectopic pregnancy, placenta accreta, and hydramnios fall into different categories of potential pregnancy complications.

A woman is admitted with a diagnosis of ectopic pregnancy. For which action would the nurse anticipate beginning preparation?

immediate surgery Ectopic pregnancy means an embryo has implanted outside the uterus, usually in the fallopian tube. Surgery is usually necessary to remove the growing structure before the tube ruptures or repair the tube if rupture has already occurred. Bed rest will not correct the problem of an ectopic pregnancy. Administering a tocolytic is not indicated, nor is internal uterine monitoring.

Some women experience a rupture of their membranes before going into true labor. A nurse recognizes that a woman who presents with PPROM has completed how many weeks of gestation?

less than 37 weeks Preterm premature rupture of membranes (PPROM) is defined as the rupture of the membranes prior to the onset of labor in a woman who is less than 37 weeks' gestation. PROM (premature rupture of membranes) refers to a woman who is beyond 37 weeks' gestation, has presented with spontaneous rupture of the membranes, and is not in labor.

A client with hyperemesis gravidarum is admitted to the facility after being cared for at home without success. What would the nurse expect to include in the client's plan of care?

nothing by mouth

A woman in labor has sharp fundal pain accompanied by slight vaginal bleeding. What is the most likely cause of these symptoms?

premature separation of the placenta Premature separation of the placenta begins with sharp fundal pain, usually followed by vaginal bleeding. Placenta previa usually produces painless bleeding. Preterm labor contractions are more often described as cramping. Possible fetal death or injury does not present with sharp fundal pain. It is usually painless.

A woman in week 35 of her pregnancy with severe hydramnios is admitted to the hospital. The nurse recognizes that which concern is greatest regarding this client?

preterm rupture of membranes followed by preterm birth

A client in her 38th week of gestation is admitted into the labor and birth unit with painless bleeding from the vagina. The client is diagnosed with placenta previa. When reviewing the client's history, which factor would the nurse identify as a risk factor for placenta previa?

previous cesarean birth A previous cesarean birth is a risk factor for developing placenta previa. This is due to the damage caused to the endometrial tissue. Multiparity, and not a primigravida status, predisposes to placenta previa. Maternal age over 35 years, and not just more than 30 years, is considered another risk factor. Placenta previa is more common among those living in high altitudes not among those living in coastal areas.

A client with preeclampsia is receiving magnesium sulfate. Which nursing assessment should be ongoing while the medication is being administered?

respiratory rate The level of magnesium in therapeutic range is 4 to 8 mg/dL. If magnesium toxicity occurs, one sign in the client will be a decrease in the respiratory rate and a potential respiratory arrest. Respiratory rate will be monitored when on this medication. The client's hemoglobin and ability to sleep are not factors for ongoing assessments for the client on magnesium sulfate. Urinary output is measured hourly on the preeclamptic client receiving magnesium sulfate, but urine protein is not an ongoing assessment.

When assessing a pregnant woman with vaginal bleeding, the nurse would suspect a threatened abortion based on which finding?

slight vaginal bleeding

When assessing a pregnant woman with vaginal bleeding, the nurse would suspect a threatened abortion based on which finding?

slight vaginal bleeding Slight vaginal bleeding early in pregnancy, no cervical dilation, and a closed cervical os are associated with a threatened abortion. Strong abdominal cramping is associated with an inevitable abortion. With an inevitable abortion, passage of the products of conception may occur. No fetal tissue is passed with a threatened abortion.

A pregnant woman is diagnosed with abruptio placentae. When reviewing the woman's medical record, the nurse would expect which finding?

sudden dark, vaginal bleeding The uterus is firm to rigid to the touch with abruptio placentae; it is soft and relaxed with placenta previa. Bleeding associated with abruptio placentae occurs suddenly and is usually dark in color. Bleeding also may not be visible. Bright red vaginal bleeding is associated with placenta previa. Fetal distress or absent fetal heart rate may be noted with abruptio placentae. The woman with abruptio placentae usually experiences constant uterine tenderness on palpation.


Conjuntos de estudio relacionados

Multiplication Strategies: Array's, Repeated Addition, Equal Groups, & Number Lines!!!!

View Set

M6 - Gas Exchange & Lung/Resp Assessment

View Set

GPHY 111: Introduction to Physical Geography #1 Test Prep

View Set

Chapter 27 Regulation & Antitrust Policy

View Set

Bio Test 2 - PP 11 Cellular Respiration (TCA)

View Set